Dead Space Jumping in weightlessness does not work. Universe Dead Space.

  • 25.04.2021

Everything is fine on Earth, 2012 did not come, the crises did not eat anyone and humanity has calmly developed, multiplied. A new religion appeared - Juvenology, which found a response in the hearts of very and very many, including those with power. The essence of religion was in reborn in the kind and immortal through death, everything was clear, but it is not clear how to do it. Supporters of the new religion were called yuvenologists, the new church itself had enormous financial and human resources for the material values \u200b\u200bof their followers.

Information about universe Dead Space. Can be obtained from two serious sources:

1. Dead Space: DOWNFALL (2008) (cartoon)

3. Dead Space 2 (2011) The game will be released 26-28.01.2011

On a distant distant planet, an obelisk of unearthly origin was discovered. Juvenologists decided that this is the divine message, Obelisk was decided to deliver to Earth. To do this, the huge crust of the planets "Ishimura" (mining spacecraft recycled asteroids and soil with planets, mining minerals) sent to the Egida planet 7. Most of the crew is replaced by youvenologists. There is a human colony on the planet.

From this point on, the Events of the Dead Space: Downfall are started to unfold, in which people are unsuccessfully fighting with the consequences of the destructive influence of the Obelisk on the mind and the appearance of people on the ship "Ishimura". People turned into terrible monsters with bone sabers instead of hands. The cartoon did not look - very scary and ends badly, all died, the worst prospects for humanity ...

While the cosmic ship reigned Kavardak, the systems began to gradually refuse, the first engines were out of order and since everything was not very bad, then the repair team was caused (A very important point, for the second part begins with him, namely the game Dead Space), then everything was alike and all soiled.

TWINKLE, TWINKLE, LITTLE STAR,

How i Wonder What You Are!

Up Above the World SO High,

Like A Diamond in The Sky!

WHEN THE BLAZING SUN IS GONE,

WHEN HE NOTHING SHINES UPON,

Then You Show Your Little Light,

Twinkle, Twinkle, All The Night ...

This song ends the cartoon and the installation of the game happens under it:

Dead Space.

And then the repairmen are flying to the rescue "Ishimur", their four, 3 units and Tetka, one uncle Isaac Clark (Isaac Clark) - we play. Repairmen arrive at the mining ship, and there is a break-torn, their shuttle breaks down, one of the monsters eat, Isaac Clark gets off the group and rushed.

The game is good, interesting, the plot is tightened despite the coronal control - Isaac does not know how to jump, slowly runs, clumsy, bad, slow down, occupies most of the left side of the screen, can not stop if the action started (for example, the reloading of weapons in the height of the battle, until he recharges Will not do anything else). And still a good game. I assume that such an control was received because of that? That was released on both game consoles. In general, the complexity of the game is largely related to the management, but I repeat - the game is good.

Office in Dead Space

Movement and inspection standard, running with the left shift key, select / open E, recharging R, Tab inventory, Map M (In the card / inventory mode 1,2,3,4, is responsible for switching the card / inventory / database ...) .

An important button B - indicates where to go on, very convenient never get lost.

Jumping in weightless action (right mouse button or Caps Lock), press the left Ctrl - if the cartridge indicator flashes red, it is impossible to move, and if not, it will turn to the place of aiming and it will become the floor.

In aiming mode, the deceleration of the object C (uses the battery status) and attracting objects f (the object then can be pulled by the left mouse button or omitted by pressing F).

Combat system.

The left mouse button is a blow of a weapon, azack is caught up and boom. Space - stupid foot in a heavy shoe, put monsters.

Shooting can only be performed in aiming mode - right mouse button or Caps Lock, while you can not run, the review is limited, the rotation speed drops (which is scary?), The shot is made with the left mouse button, alternative shooting on the space button.

Economy in Dead Space

Yes, yes, there is an economy as about as in the Biocese.

First we collect all that lies badly. Open boxes and lockers are highlighted with blue light, open on e, green boxes (in weightlessness often pull up f) - let's give the foot and select resources. To enter into all open rooms, always have a 1 power unit with you, to open rooms with a large stock of useful items.

After the death of monsters, cartridges and money remain, and the cartridges for the weapons that are in your inventory.

On the way there are shops in which you can purchase weapons, cartridges, first-aid kits, Stasis field batteries, oxygen cylinders, armored vehicles, and you can sell what you managed to collect along the way or shifting into your personal safe. The assortment in the store is originally not good, but as Izek passes the schemes for new goods and costumes expanding the list of products. In stores need to go more often leaving extra ammunition there, selling unnecessary equipment and schemes.

IN Dead Space should be worn: 2 first aid kits, 1 battery, 1 oxygen cylinder, small margin of cartridges, 4-re-type of weapons, 1 power unit.

IN safe Create a stock: 5 ADDCHES, 1 BATTERY, 1 Oxygen cylinder, excess ammunition, these resources should be taken more than enough to pass complex areas.

Everything else is sold to the store, without regret. For all the time the game had to buy only weapons in the store, power blocks, armored vehicles and cartridges (and then because before that, they did not save them economically and often sold to the store).

Priority purchases - Branches try to buy immediately after the appearance occurs 4-5 times the latter cost 60,000 rubles, strength blocks.

Weapons

Keep the weapon always charged, during the battle after the end of the ammunition, switch to the next weapon, recharging takes time.

I didn't like all weapons, but maybe I just did not fully revealed his potential.

The pulse cutter - shoots the limbs with a horizontal or vertical beam, the most first weapon, alternative shooting mode - a horizontal shot, upgrade in the direction of zooming damage, in parallel increasing the capacity of the store (up to 16 ammunition).

The pulse rifle is an automatic, an upgrade for an increase in damage, parallel with a capacity (up to 150 rounds).

Laser rifle is very strong damage even without upgrade, the small capacity of the store (5 ammunition), breaks almost any monster and even a few, upgrade to the lady and on the store's capacity, alternative mode - leaves mine.

Flamethrower - applied against small monsters running flocks, not an upgraded was replaced with a power gun, not working in vacuo.

Power gun - makes mass damage and discarded enemies, upgrade to damage, in parallel to the store's capacity (14 charges).

I didn't like the power cutter - before the shot should accumulate energy, and then shots and save again.

Truborez - did not try, it's a pity.

Upgrade weapons

Improving equipment is made on the workbenches that are scattered throughout the ship. For improvements, the power units that we noted as priority when buying in the store are needed.

The order of upgrade.

1. Pulse cutter - damage.

2. Laser rifle, power gun, pulse rifle - pumping to an increase in damage, as a result of this, other indicators of weapons will increase. The order is not important, the main consistent pumping of each weapon used.

3. Armor - pumping life, the air is not particularly required - the standard stock is enough for the passage of all sites.

Monsters of the Dead Space Universe and the fight against them

Monsters attack almost always unexpectedly, but before they start to tear your flesh they stand and grow about 5 seconds and make up the limbs, it is enough to aim, shoot legs (for a slowdown) and shoot your hand for the final victory. To kill Monster, he needs to shoot what he sticks out, her legs - slow down, hands and head for finishing.

Monsters are divided into strength on pink and black (reinforced, require more ammunition).

The appearance of monsters is accompanied by a grace of music in obligatory.

The giant tentacle that is enough and drags into the wall - shoot on the yellow plot on the tentacle.

The monster with tentacles grown to the wall, fits cocoons (infinitely) - shoot a tentacles or reject the exploding balloon with gas into it.

Large fast monster - we send to the Stasis, go from the back and rolle, two or three times it is enough.

Immortal monster (the most vile creature, meets twice, both times he has been released by a bad youthroom) - here you can only slow down the legs and your hands after that there are 10-20 seconds while it regenerates, it is necessary to run or throw a stasis in the destruction zone.

Passage

In the network you can find many references to passage, but for the most part they are the same. In general, the passage of the game is not difficult, you will always help you where to go, the button in will indicate the direction, but there are nozzles when you just do not know what to do, here we will talk about them.

1. Battle with an immortal monster. In about the 5th episode, we face this creature, go into a large room, and there this bug, a bunch of other monsters and doors are all closed. Calmly. We immobilize the immortal, interrupt ordinary monsters, run, repeat the procedure and your collar will take care of the opening of the door. In the same episode, we need to finish it in the Crycamer - shoot legs when you crawl into the camera to throw the Stasis, run into the room with a remote and freeze. Here he is reptile:

2. Jogging under meteorites, hiding behind the couches while there is a meteor flow, in the minutes of the clutch by going to the next shelter.

3. The asteroid to which the lighthouse needs to be placed. We climb on the asteroid and on its surface to be treated into space, they calmly put the lighthouse and there are two holding generators there (we find them by key c).

4. The 10th episode (here in my opinion it is necessary to make money on a costume 60,000) again the immortal monster came up, and we just had to move the bedrooms, we are immobilized, we throw in the Stasis, we move the beds, go. To destroy a monster, lubricate it to the shuttle engines, immobilize, throw the Stasis and burn the engine test.

The game is passed, bad defeated, the land is saved, Isaac saved, everything is fine ...

But no. And here begins

Dead Space 2.

it turns out on 01/26/2011 in Russia, so the translation will be at the height, control, atmosphere, equipment will probably retain, but for now, videos:

No related posts

.
2. Intensive therapy (Intensive Care).
3. Orbital Correction (Correction).
4. Death is inevitable (Obliteration Imminent).
5. Lethal Devotion.
6. Hazardous impurities (Environmental Hazard).
7. Step into emptiness (INTO THE VOID).
8. Find and save (Search and Rescue).
.
.
.
.

  • Questions - Answers .
  • Walkthrough

    9. Dead on arrival (Dead ON Arrival)
    Dead Space How to go. Chapter 9.

    Officer Hammond proposes to pick up spare parts from the Wailor warship of the warship and repair the local shuttle. Then the connection disappears, communicate with Kendra about the same.


    Ore Storage

    We go to the zone of weightlessness. At the bottom of the huge flaps we hit 6 fuses with legs, they are located on both sides. After that, the gate will open, the air will begin to be consumed. We need to catch flying round rose balls, and throw them into space. One of them is hidden on the wall behind the round platform. Near the open flaps are two panels, where you can replenish the oxygen stock. After throwing all the balls, we enter into a deepening in the wall, get on board the Veilor ship.


    Cargo Bay.

    A very fast necromorph will run away from us in the corridors. We go to the site where the kinesis needs to move magnetic boxes to pass forward. In lateral deadlocks on the right and on the left you can find cartridges, if you move unnecessary boxes. After these obstacles, two fast monster will attack immediately. In battle, they do not differ from ordinary enemies, they only quickly hope to us.

    Next again narrow corridors. The first battery does not take from the wall, and you can. In the last room, the elevator battery fails, mice will appear and will begin to revive the corpses. We understand with the enemies, insert any other battery, climb the elevator.

    In the elevator cabin, Dr. Terrence Kane will contact us. He will tell that the reason for all the troubles is the artifact "Marker", which was discovered on the planet and delivered to the Ishimour. To stop the invasion of necromorphs, you need to return the "marker" to the place. If we manage to repair the shuttle, then it should be used for this case, and not evacuated.



    Fight with a divided monster, a little further - with Kamikaze. We take from the wall of the power node (23). At the turn we find a shooting gallery, we can practice in shooting. It is necessary to shoot in red armed mannequins, and not shoot in unarmed blue. Endless learning cartridges, modem weapons apply any, but the standard gun is preferable. Total can pass 5 tests:

    1. Kill 10 enemies, 1000 points. Award: Additional Cartridges.

    2. Kill 12 enemies, 1200 points. Reward: Pistol cartridges.

    3. Kill 14 enemies, 1400 points. Award: A middle aid kit.

    4. Kill 16 enemies, 1600 points. Reward: ruby semiconductor.

    5. Kill 20 enemies, 2000 points. Award: Power Node (24).


    Infirmary

    We are going to Lazaret, here a faulty laser scalpel, the rays of which are deadly dangerous. We slow down the laser by the Stasis, quickly run by. On the way to the left of it you can take a large first-aid kit (LARGE MED PACK).

    In the barracks in a dead end, we take the power node (25). In a double corridor, all sorts of monsters will be attached, while the door does not open.


    Engine Room

    We get to the machine compartment, the ships engine see the windows. Raised on the elevator, the powerful stream of fire occurs from above. We need to hide at one of the cylinders, shift it with a kinesis, and to move forward so much. If the cylinder is stuck, we move it. On the way you need to destroy 3 fuses on the pillars. After you need to go back, and pull the cylinder on yourself, which is much more difficult. We pass forward on the other hand, also destroy 3 fuses. After that, the engine will stop. We take singular converter in the center.

    Go down. For the glass we will see the Hammond came running, but it will immediately kill the red brut. This monster has more health than ordinary healthy, but kills the same way. For the victory over him we get diamond semiconductor For 25,000 credits.

    Wairor's ship began to collapse, quickly choose from it. Along the way, we kill a couple of ordinary monsters. Get to the trolley.

    10. Recent days (End of Days)
    Dead Space Passage. Chapter 10.

    Candra reports that I found the location of the shuttle on the ship. But before his launch, you need to collect navigation cards to get to the ground. Search them will be in the residential compartment of the station. In the lobby we see many killed, as with a sacrifice, and near the strange figure, obelisk.


    Sleep Bock C.

    Right enter the open door, find the shower lEVEL 5 SUIT). We can immediately return and buy this costume for 60,000 credits. Further in a dead end, a mini-G Basketball mini-G game. Inside the gym with low gravity lies 1st navigation map. Here we can play this game by turning on the panel on the first platform.

    Z-Ball rules: Of the four holes, the ball flies ahead, we need to catch it and throw it back into any hole. But if we do it right away, we will get only 1 point. When the next ball crashes, several platforms light up, first jump on them, each visit will double the glasses, and after that you can catch and throw the ball back, earn 2, 4 or 8 points. For a minute you need to dial from 12 to 40 points, depending on the level. There are 6 levels in total, for each of them we will receive prizes from lockers:

    1. Just throw the balls. Award: Fuel flameth.

    2. Holes slowly close. Award: A middle aid kit.

    3. Opened only one hole. Reward: a rayan battery.

    4. One hole opens for a while. Award: 3,000 loans.

    5. Holes are opened in turn. Reward: ruby semiconductor.

    6. The holes are opened in turn faster. Award: Power Node (26).


    Mess Hall

    We rise to the second floor. For the glass we see a crazy doctor, he brings a sacrifice - kills a man. We enter the cafeteria, but I do not have time to catch the doctors. We descend under the cafe, we kill two long monsters, from the wall at the table we take the power node (27). Near the woman who took the wall, take key carriage. Climb and go to the next room.


    Sleep Block A.

    In the corridor, we kill the necromorph in the wall. We go on the following floor through the rest room. We will grab and drag the tentacle, shoot in yellowightened. At the end we reach the panel, on the left of the kinesis we move the magnetic lever, it will open the doors on the floor.

    Sleep Block B.

    We enter the depressiveized residential compartment, where oxygen is consumed. There are many monsters, but we do not pay attention to them, there is little time. It is especially not necessary to kill thick monsters, of which leeches will appear, and the flamethrower does not work here. Two times we turn to the right, in a dead-end room we find the power node (28). In the central hall between the rows of Kubinkov will find 2nd navigation map.


    Sleep Bock C.

    We return back to the hall, from there go to the compartment "C", now with the help of the key we can open the door to the bedroom zone. On the way I will see a video of the speech of the doctor.

    In the bedroom on the right we can spend the power knot to open the door, for it various cartridges and a middle aid kit. Kinezis move the bed to go to the cache behind them: I move the bed at the wall, then the bed is across. Entering the intermediate part, we move back the transverse bed, we move two more beds at the right wall, we leave.

    In the cache, we find many corpses and a laughing girl, who immediately suices. Right from it take 3rd navigation map.

    Quickly come back. We will begin to pursue the regenerating monster. Doors will close in the intermediate room, and we will have to run some time with circles from an invulnerable monster until Kendra hack the door. In the lobby, sitting on the elevator.


    Security Station

    We find refuge of Dr. Kane, who helped us. Communicate with him through the glass. He misses us to the direct school.

    Executive Commons

    In the large Red Hall of 3 fat monster. We search for 4 siderooms, in one of them Audio Draw Captain, in one note of the assistant, in the rest - ammunition. We enter double doors.

    Executive Shuttle Bay.

    We get to the starting area of \u200b\u200bthe shuttle, we enter into it, install navigation cards. Dr. Kane asks to wait until he gets here. While we go to the side room and test the engine there.

    But before the doctor, the regenerating monster will turn here. To kill this boss, lure it on the site opposite the shuttle exhaust pipes, shoot him legs, freeze the stasis. Quickly run into the room and turn on the engine again. Monster will remain only ashes and power node (29).

    Dr. Kane will come running and get inside the shuttle. In the room on the second panel open the valve, the shuttle goes down. We go back to Monorails on foot. On the way back, let's see the video like a mad doctor Mercer voluntarily turns into necromorph. We pass to the next room, we kill the bat and mutating the doctor.

    11. Alternative solutions (Alternate Solutions)
    Dead Space. Video list. Chapter 11.

    Cargo Bay.

    Kane by radio will explain that we must find black and red obelisk in the cargo compartment, and download it to the shuttle. Go down to the floor below, first go around the square track all the location, activate the panel. In the far corner on the wall of the power node (30).

    In the center on the conveyor belt we find the obelisk in a glass box. It is necessary to move the obelisk in the conveyor using a kinosis. We are prevented by necromorphs, and when we go out on the central tracks, we are attacked by the giant tentacles, first need to scare them, and then move the cargo.


    FLIGHT LOUNGE.

    We return to the room where the first arrived at the station. If we go on the corridor, where they ran from the first monster, finding a dead end ruby semiconductor and cartridges. Then we go out into the spaceport.


    Hangar Bay.

    Shattl Kane will arrive in the hangar. The loading system does not work, we need to manually download the obelisk to the shuttle. We go to the room to the right, turn off the gravity on the panel, jump down under the ship.

    Secret. In the hangar, we look down to the ditch between the bridges, the Peng statuette is lying on the bottom, we can pull it with a kinesis module and pick up. It is unique, the most expensive thing in the game, we can sell it for 50,000.

    We move the obelisk further, but here there are two fork on the rails, turn them so that the occasion is right. At the first crossroads wech the obelisk in the side deadlock, turn the rails on the left side, move the load to the end.

    We return to the panel on the balcony, we turn on the gravity, the obelisk will plunge. We run to omnounced ladder in the shuttle. But right in front of us Dr. Kane will be killed, and the shuttle will fly away without us. All this arranged Kendra, she admits that was a secret agent, and her goal was to get obelisk. Obelisk himself is a copy created by people for experiments.

    Nicole will come to contact. We rise to it on the top floor of the control room, we take diamond semiconductor. On the screens around the interference and extraterrestrial characters. Nicole will tell you that the shuttle can be returned to the Angar by emergency remote control. The shuttle is really returning, and Kendra catapults on the planet, frightened by our revenge. We enter the returned ship and fly yourself.

    12. Dead Space (Dead Space)
    Dead Space (2008). Chapter 12.

    Landing Pad.

    On the planet we land on the special platform of the earth's colony. All people are killed here. We enter the building, inside there is a semiconductor and everything you need: shop, workbench, saving. In the side locked room, two monsters are locked - bats.

    Under the shuttle, we shift the platform, we load onto the obelisk, we move along the rails. Near the door lacks the battery, go to look for her inside the building, then continue the way.


    Airlock A, Airlock B

    We move the obelisk further, we will interfere with the whole hordes of necromorphs, but we can constantly use the Stasis, there are many panels here for its replenishment. In the block "b" in the right far corner there is a door opened by the power node. In some places you need to carry out obelisk under the bridges, for this, press the panel, freeze the bridges by the Stasis.


    GRAVITY CONTROL

    On the way on the right there are two enemies in the wall. Bridges do not rise, you need to go into a large pipe and turn on electricity. On the way back all the devices will turn on and we will have to pass through moving blades, slow down their stasis. When we return to the main way, it is necessary to fight with the brutal, for the victory over it we get a power knot (31).

    In the next small compartment on the right there is a power node (32).


    Pedestal Chamber.

    We go out on the open surface of the planet. On the way we have a dozen monsters and three tentacles from the cliff. We clean the way and we dotion the obelisk to the cliff.

    The alarm is triggered, running into the glass room on the left. We are locked, and the kendra appears outside, it moves the obelisk back to the shuttle. Also, Kendra throws us a complete video communication from Nicole, in which it sustates. It turns out that the real Nicole has long been dead, and all that we have seen is hallucinations caused by Obelisk. (In the game hid a hint about it: from the first letters of the names of Chapters Dead Space in English there is a phrase: Nicole Is Dead - Nicole is dead).


    Living Space.

    Early surrender. From the glass room we go to the side corridor, overgrown with biomass. On the way, take the power node (33). We return to the first room, last procurement in the store, we spend the power knots, we take more cartridges and first-aid kits. We go to the shuttle.


    Boss: Hive Mind (Mind Roy)

    Kender tried to hide on the shuttle, but at the last moment she was killed by the tentacle of a giant monster. We remain with this monster one to one. This is the leader of necromorphs, being on this planet, he telepathically controlled all monsters.

    1 phase. The boss will hit tentacles from above. We constantly look in his direction, run away left-right. After hitting the boss will bring her mouth towards us. Around the grazing, he has 5 yellow thighs, you need to shoot on them. It is better to spend powerful slow weapons, and the rapid base pistol is left for later.

    2 phase. When they shoot three any outflow, the monster grabs us with a tentacle. In the remaining two outflows will have to shoot on the weight, hanging up the legs. If we do not have time, the boss will swallow us.

    3 phase. Taking yellow dots on time, we remain alive, and fall on the platform. The boss does all the same, but now after a few blows, the Röbra will be revealed, and 5 yellow hearts hang inside them. We use more accurate weapons - the initial gun, gradually shoot the heart.

    When all the hearts are shot, a huge monster will fall off the cliff. Sit down to the shuttle and leave the planet. Inside the last time we watch a video from Nicole, and fly to the ground.

    Congratulations! The whole game is passed!

    For the passage we get the following bonuses:

    Military suit (military armor);

    Backstory Logs (Text messages with the history of the plot);

    50,000 Credits (money from the very beginning);

    10 Power Nodes (10 power nodes);

    Impossible Mode (the maximum complexity opens).

    Questions - Answers

    Question: How to install a lighthouse on an asteroid? What to do with an asteroid? (in the game Dead Space)

    Answer: The asteroid holds 4 beams, we turn off the two of them inside the ship, two more - outside. Only after that we can put the lighthouse. Read more described in chapter 7.


    Question: How to jump in weightlessness? (in the game Dead Space)

    Answer: In the first part of the game, you can not fly in weightlessness, we can only jump over the walls and the ceiling. Aims into the appropriate wall (clamp the right mouse button), we make a jump (press the Ctrl key).


    Question: How to use Stasis? (in the game Dead Space)

    Answer: Aims (clamp the right mouse button), we release the delayed charge (key C).


    Question: How to enable, change the Russian language? (in the game Dead Space)

    Answer: In the licensed version of the game there is no Russian voice acting, only subtitles. To enable subtitles, in the main menu, select the "Settings" stitch - "Sound" - "Subtitles: Enable".


    Question: How to pass the meteor rain? (in the game Dead Space)

    Answer: At the end of the 4th chapter. In order not to die, before the start of the meteoric rain, you need to get up for high shelter, close to it, and wait for a dangerous moment. After that, quickly overlook the next shelter, and so until the end of the way.


    Question: How to remove the power castle? (in the game Dead Space)

    Answer: The game has official storage facilities, locked on the power lock. To open them, you need to spend 1 power node. Nodes can be found in secrets or buy in game stores for 10,000 credits.


    Question: What part is better? (in the game Dead Space)

    Unusual game with interesting ideas - from Electronic Arts? A year ago, I would be very surprised, tell me who about it. The gaming publisher, who from year to year released dozens of sterile sports simulators, no lonk the addition of additions to Sims and properly fed the market on kinolizses, was associated with anything, but not with innovation.

    However, as of the state of affairs, in the fall of 2008, it is true. This is an editor and a generator of new ideas, unusual worlds and atypical games. You only listen - "Sci-fi horror from Electronic Arts" ...

    Breakdown

    Zaguka in Dead Space is a banal. The huge spacecraft "Ishimura" class "Ripper Planets" - he literally destroys the planets in the process of mining - one finely stopped to contact. To find out the situation to "Ishimur" send a small repair shuttle.

    On board, among other crew members, there is someone Aizek Clark, an ordinary engineer, on the role of the rescuer is completely inappropriate. After the tight fit on the "Ishimuru", it turns out quite quickly that the radio collement and emptiness on the moisture deck are not caused by non-malfunctions, but by terrible monsters that flooded the ship. The crew of the repair shuttle is partially eaten, and the survivors run away at different angles, then together with joint efforts, communicate mainly through radio communications, try to get out of the damned ship.

    Here and "Ishimura" - suspiciously silent. The Kellion crew does not know that they are waiting for them soon.

    Despite the standard beginning, the game quickly unfolds in full power. Overboard merciless space, around neither the soul, and Isaac together with the player gently picks up the feeling of loneliness and despondency, which is only enhanced by rare radio sessions. This is not a bright future, where the careless spaceships are furious by the expanses of the smiling universe. Even if you drop flickering light and bleeding corridors with confused corpses, the overall design of the ship and the environment sets the autumn mood.

    But they scare not empty corridors, half a dream, sudden attacks of monsters and traces of blood on the walls and sex. Surroundings only sets the mood, the real strike applies sounds. It is strange and frozen sounds that be it incomprehensible screams or a crazy opposite crushing, make you nervous and look around on the sides. They do not force the situation, warning about future danger. No, it was too simple. Sounds hint that "Ishimura" is far from a safe place and should not relax.

    The developers are sound, which is often supported by some unpleasant, but not a fatal event, hint that something terrible will happen now. Internally make it on the worst, but nothing happens. The fear charge does not receive an emotional splash, and after a while you begin to fear constantly, literally every minute of the game. This incredible psychological attack brings abundant fruits.

    When monsters appear, emotions and adrenaline emotions, especially in the first phases of the game, it is so knocked that sometimes the hands are naturally trembling. Voltage - Permanent satellite Dead Space; Waiting and semi-rigging, and not the crushed mouth of monsters make nervously look around. At the same time, there is not a single safe place in the game. Even indoors with constants sometimes live monsters.

    Cruel Space

    Do not relax and contracted with monsters. Necromorphs, which is exactly the name of the alien form of life, is very resistant to damage, and the only effective way to combat - cut off important limbs. It falls as a plasma cutter and other construction tools in the literal sense to gut the monsters.

    You will have to periodically go into space, where Monsters are also found.

    The cartridges are always lacking, it has to think through each shot to not stay with empty pockets in the midst of the battle. Savings are the cornerstone in the game process. And the point is not limited to one cartridges and first-aid kits. Weapons and equipment need to be improved, and the power blocks are material for improvements - always lacking. Therefore, you have to choose that you really need and where to invest in the first place. Spare parts will have a maximum of three models.

    Unfortunately, the developers could not withstand a hard pace until the end of the game. Obtained two thirds levels, you significantly improve the weapons, and the fights will become much easier. At the very end of the game, the monsters will no longer be a problem that somewhat dilutes the atmosphere of tension and fear. However, we have to do not only do not only shoot necromorphs. In Dead Space, fully unusual gaming situations and interesting tasks.

    The greatest surprise carry out exits into open space. There is nothing special about it, but how these situations are filed. Suddenly all the sounds disappear, and we only hear the head of the heart and deep breathing of Isaac. Zones with zero gravity were perfectly successful, where the walls and ceiling become the second floor, and the character famously jumps in any direction on tens of meters. True, in the last two levels, the game rolls to the banal extermination of monsters and the thrust pushing of the cart through the entire card.

    In the game, even a backpack is made unusually - in the form of a holographic menu.

    The picture in the game is surprisingly alive, although sometimes there are muddy textures. It can be seen that designers and artists worked over the common style of the game. Smooth and unhurried Aizek movements, excellent animation during neighboring kits with monsters and in interactive scenes. In Dead Space, the interface is built directly into the game: Health Indicator - Stripe on the back, the number of cartridges - right on weapons, level map, task menu, backpack - are displayed in the form of a hologram and at the same time the game is not afraid.

    The plot, despite the seeming simplicity, will surprise unusual turns several times. But the scenarios, taking a fairly cheerful pace at the beginning of the game, sometimes too sharply and meaninglessly cut the storylines. Especially got the characters. But the heroes in Dead Space can be counted on the fingers of one hand.

    It was seen that the developers tried over the universe of the game. Animated comics before the exit, a full-length cartoon after, it's even threatening to walk to a feature film. Nevertheless, with close attention, logical inconsistencies in the plot are immediately becoming visible, and additional materials - such as the cartoon Dead Space Downfall, do not give answers to questions, but only add new contradictions.

    However, these inconsistencies cause only surprise, but not irritation. The plot in Dead Space is more like a framing of surprisingly accurate and careful citation of primary sources, a crazy atmosphere and an derived playing process. I want to believe that contradictions will be eliminated in other games on this universe. If so, of course, will appear ...



    Despite a number of miscalculations in the game mechanics and the plot, Dead Space deserves close attention and sincere love / fear. Heart captivated familiar and carefully cited elements from "Aliens" and "Something" (this is, by the way, already kininelateka), an unusual approach to intimidation and an incredible atmosphere of panic, unknown and the future. Have you already decided? Then get ready for a fascinating journey.


    Dignity LIMITATIONS
    Fascination
    9
    battles with monsters, a variety of game situations, the atmosphere of fearcloser to the end of the game, the fight becomes too simple, the play situations are too often repeated in the last levels.
    Graphics
    8
    pleasant models, special effects, level design, animationtimes muddy texture
    Sound
    10
    sounds and music skillfully creating an atmospherenot
    Game world
    8
    at first glance, a rustic, but strong and with unexpected turnslogic inconsistencies in the plot, senior attitude to characters
    Convenience
    9
    management, optimizationindividual bugs, inconvenient to play not widescreen monitors

    The main skills of the character

    Isaac can move in two modes - step and running. In addition to speed, they do not differ anything. It is clear that you need to run when you save from monsters, and walk - all the other time. But the character can run sideways. It is impossible to shoot, but so you can dodge the shots, especially when the opponent is far away. The camera in Dead Space is directed in such a way that you can clearly see what happens in front, and it is almost not clear that it is going on on the side. Therefore, more often twist your head so as not to blink necromorph.

    If you do not have time to get out of vacuum before oxygen ends, it will happen exactly that - Isaac will fall and die.

    When you fall into zero gravity zone, the rules change a little. You can also walk and run, but now not only on the floor, but also the ceiling with the walls. To quickly move from the wall on the ceiling, aim in the place where you want to get, and press the jump key. Isaac will pushes the surface and flies through the entire room.

    Thus, you can simply quickly move or escape from monsters by flight. If for some reason Izek can not jump, the cartridge indicator flashes in red, and you will hear a characteristic sound. Remember that monsters in weightlessness are also perfectly crawling along the walls and the ceiling, so you have to actively twist your head to determine their position and do not stop the attack.

    Sometimes you have to go out into open space. At the same time, an oxygen indicator appears on the back of the character. When he is over, Isaac dies from choking. Oxygen will automatically recover when you get to the room with air. You can also refuel from special devices or carry with a spare balloon with air. Know if you found a container with oxygen, it means that soon in open space.

    If the monster came close to too close and you do not have time to shoot, then it can be perfectly attached to a fist. From this he will fly back, and you can shry or run off. Handpoint does not cause serious damage, but stuns the enemy. Therefore, use it, only to push the necromorph. If the monster crawls on the floor, you can hit it with your foot. Thus, it is convenient to finish the wounded wounds and put pressure on small monsters, pre-braking them with a station.

    If you got lost, use the navigator to find the road.

    Sometimes the monster does not just hit with sharp as a razor, the growths, and jars on the character and begins to bite for different parts of the body. To reset the necromorph, click on the key specified on the screen. Sometimes Isaac does not just leapes the enemy, but immediately kills him without your help. It usually works against small monsters. However, such attacks are very harmful to health, so adherent monsters need to be afraid of fire.

    And of course, the most important aspect in the game is the ability to handle weapons. It would seem what can be difficult here? Fixed and shoot. So it is so, but at least at first you will have a deficit with cartridges, so you need to shoot exactly to apply the maximum damage. Also, each weapon has an alternative shooting mode, so sometimes it is necessary to think about how to charge on necromorpses in a particular situation.

    In Dead Space, the level navigator is elected. You can open a shared map, thoroughly study it at different angles, increase or decrease the review. But constantly open the additional menu to determine where to move on is quite uncomfortable. Therefore, you can press the quick navigation key. From the device on the hand of Aizek will shoot a blue beam, which will show where to go. If a few goals, the navigator will lead to the nearest.

    Who are necromorphs?

    Necromorfs are not a revived dead and not form of mutation. This is a foreign form of life that uses dead bodies as building material. Necromorphs are not conscious and are created exclusively for murder. After all, the more corpses, the more fellow. Therefore, monsters are extremely active and completely devoid of instinct of self-preservation. They are just the tools that the powerful alien intellate leads.

    Stasis and Kinesis

    Sometimes you need to alternate your skills to solve the problem. For example, first slow down the Rotor Stasis and draw the kinesis.

    The stasis module allows you to temporarily slow the necromorphs and the mechanisms needed according to the plot. Does the door opens and closes? Slow it with a station and run past. Against Monsters Stasis at all chopper-corrupt. What you use when the opponent stands still, it seems to me, and so understandable. You can accurately aim for a shot, run off to a safe distance, bypass the monster from the back or delay it until you figure it out with other opponents. But the Stasis has a habit to end, so look for chargers or spare batteries.

    Having received a kinesis module, you can move items at a distance. This skill is usually used to solve non-hard tasks: disassemble the blockage in front of the door, pull the trolley to the site, etc. Also, using the kinesisse, it is convenient to collect items. The cartridges lie on the shelf, but do not climb her hands there? The box is lying down, and I don't want to go to her much? So tighten the kinesis.

    At times, the kinesis is a good weapon. The module can not only tighten the items, but also run them with a terrible speed. Therefore, sometimes, especially when necromorphs are going to the crowd, thumps well in monsters of barrels with flammable. Capacities with flammable you can easily learn on anxious red color. With the help of a kinesis, you can raise some massive object and overcoat the road to monster. True, it should be done only for entertainment when Isaque does not threaten danger.

    Battle with monsters

    Each type of necromorph requires a special approach, so we will analyze specific battle tactics in Bestiary. Now let's talk about common things.

    Slow down the enemies to the Stasis so that they do not attacked and it was easier for you to go.

    The first thing you need to know is weak and strengths of each type of weapon. The pulse rifle shoots quickly, but strikes a small area, while the plasma cutter causes serious damage, strikes a large area, but it suffers with rapidity. Therefore, the first important point is to choose a convenient combination of weapons. It is advisable to make as quickly as possible so that you immediately do the improvement of the trunks.

    The second, what you do not need to forget - the way of movement. Given that the monsters are a lot and they suddenly jump out of different angles, and sometimes they fall from the ceiling, go slowly and carefully. You have nowhere to hurry, in monsters during the game, look at the same time. Going to the turn, twist the camera in order to see what is happening there. Sometimes you can notice necromorph before he sees the main character.

    Try to use the architecture of the premises with the maximum benefit for yourself. For example, you can decompose the kegs with a flammable one of the big bunch, if you know that the horde of necromorphs will soon run. Try to look for secluded corners where the rear and flanks will be covered. If the enemy goes the crowd along the narrow corridor, it is only for advantage. Necromorphs will interfere with each other, and you can freeze going ahead, creating an excellent stopper and target for firing.

    Ancient as a world of lubrication tactic is superbly working in Dead Space. But there is one factor that you should not forget about. Monsters can climb into ventilation so that it is suddenly unexpected behind the back. It must be considered, but do not need to be afraid. In addition, if necromorphs begin to jump out from ventilation, it is only for use. Stand in place and shoot monsters one by one.

    Rippers

    Rippers, probably the most common monsters in the game. Sparkling, with human hands sticking out of the abdomen, and two huge paws, growing from shoulders. At first, they slowly poured to the victim, but, going closer, make a sharp jerk. First of all, shoot them legs to slow down the movement, and then cut off the razors to finish the monster.

    Not too dangerous enemy. The main thing is to prevent surroundings with these creatures, then there may be problems. Rippers are two types - ordinary and improved. The last externally differs only by the color of the skin (or what they have ...), but much more lively. You will have to spend much more shots to cut off limbs. The tactic of the battle does not change - first legs, then hands-blades.

    Trade and improvement

    Cartridges and equipment can not only find, but also to buy. Location of stores can be viewed on the map. Initially, the range wants to leave the best, but, looking for new schemes, you add objects for sale. Some items can only be bought in the store. For example, armor and weapons.

    Armor is five levels, plus the sixth prize, which opens when re-passing. You start the game with first-level armor. The second-level armor can be bought at the first level for 10,000 credits. The third-level armor will be available in the fourth level and costs 20,000 credits.

    Do not forget to buy new armor as soon as you find a new scheme.

    The fourth-level armor will have to wait until the seventh level, and it will cost 35,000 credits. The fifth level armor you will receive 10,000 credits. Finally, the sixth level armor can be bought by re-passing for 99,000 credits.

    In the store you can buy cartridges. For this, as, however, for any other product, you first need to find a scheme. But I believe that you can safely do without these extra spending. The ammunition is enough at reasonable consumption with interest, and at the end of the game they often do not even fit in the backpack. If not enough, you need to fight over the increase in shooting accuracy. In addition, prices bite, and if they constantly acquire the cartridges, there is no longer enough money.

    By the way, about birds. At the beginning of the game, you will experience the shortage of everything, but gradually reserves will grow, and there will be no place in the backpack. In this case, you can shift things in the safe through the menu in the store. If stocks are too abundant, it makes sense to sell part of the items. Do not forget that most often you will find ammunition for the weapon that you have in your hands. Therefore, be sure to leave in a safe or, which is even better, sell trunks that do not use.

    You can buy first aid kits, oxygen and a stoxed battery in the store, but again I can repeat, you can easily find them at the level. The stationary battery restores the energy of the stasis module. I do not advise you to keep more than two in the backpack. And the best way to do one, good enough of stationary recharging.

    The first-aid kits are three species - small, medium and large. Little restores one health division, medium - two, big - everything and immediately. I had enough to carry two medium with me. Large medpacks are relevant only at certain points of the game. Oxygen cylinders restore air reserves and there are three species - small, medium and large. You will usually find them before entering space.

    One ways to earn - sell unnecessary items. Also, loans are scattered throughout the level, hidden in lockers and from time to time fall out of monsters. There are special items - chips that serve exclusively for sale. They most often lie in secluded places or cache. The gold chip costs 3,000 loans, ruby \u200b\u200b- 10,000 credits, diamond - 25,000 credits.

    Improve your weapon using the workbench.

    The last object in the store on the list, but not significantly - the power unit. It is used to improve weapons and equipment. But there are rooms in the game, open which you can only spending the power unit. In such a room there are many valuable items, which you usually be able to buy a power unit and stay in the plus. But it is better to first record the game, and then check. Each block costs 10,000 credits, and although they are regularly found at levels, in the presence of free funds it makes sense to purchase extra. Improvements are held on the workbench.

    Each weapon or equipment has its own scheme of improvement, and often to get to the desired damage plus three, it is necessary to invest not only in side improvements, for example, an increase in the capacity of the clip, but also into empty nodes that have no advantages. The power blocks are always lacking, so you can improve the maximum of no more than two stems and part of the equipment.

    We will talk about improving weapons in a separate chapter. Here we consider only the equipment. You can improve your health four times, the maximum result is plus 100% to the initial indicator. But for this you have to invest in eight empty nodes. Also on this scheme is oxygen. You can extend stay in space up to 120 seconds. But, honestly, I do not see much sense to invest in this improvement.

    The kinesis module can increase the radius to 12 meters. For this, three power blocks will be required. There are no empty nodes in the diagram. Useful, but not critical improvement. Therefore, you can not hardly hurry. Whereas it is more useful to invest in the Module of the Stasis. You can increase the maximum amount of energy by 75%, as well as extend the deceleration time to 30 seconds.

    Each parameter will require three power blocks, and you will also have to spend five blocks to empty nodes. Costs, of course, are decent, almost like health. But the benefit is huge. You will be able to slow down more enemies, they will stay no longer helpless, and it will be necessary to recharge less frequently, which saves time.

    Where to look for subjects

    Search for items in the game is not as simple as it may seem first. Due to the lack of normal tips, it is at first difficult to understand which objects are interactive, and which are only interior items.

    Of course, items fall out of monsters or simply lie on the floor. They clearly glue from afar, and when you come closer, an image will arise with the name of the thing. Items can be selected with your hands or pushing the kinesis.

    Also, items lie in high and narrow lockers (both in changing rooms) and low containers. Not all boxes and containers can be opened. Closed is on the red strip-indicator, at the open - white. Finally, you can get ammunition and equipment, breaking the leg and driving a small box with a green round light bulb on the surface.

    Weapons

    In the game seven types of weapons. The first is a plasma cutter - you will receive free at the first level. The rest will have to buy in the store. It makes no sense to buy everything. In quick access cells, you can accommodate four barrels. Improvements will be enough for a maximum of two, well, maybe for three types of weapons. In addition, as practice shows, three guns are usually used, and the rest are lying out.

    What weapon to choose? A matter of taste. Each gun has its strengths and weaknesses, and they all complement each other. Therefore, to say that the plasma cutter is best, and the rest of the models can be thrown out, it would be not true. I will try on the basis of my own experience describe the pros and cons of each model. What to choose, solve you. But remember - better when the theory is supported by practice.

    Plasma cutter

    Plasma cutter - the first weapon you will get in the game. It is issued for free, while other needs to be bought. The cutter shoots a fairly wide ray of energy, which causes decent damage. True, it is only at the first levels, then you will begin to feel the lack of power, and have to be in advance. Fully improved cutter will remain relevant weapons until the end of the game.

    Alternative fire - Turning the trunk. That is, you can shoot both horizontally and vertically. It is very convenient when you need to precisely cut off the limbs. The legs pulled off, the barrel turned, took up the handles. For example, a plasma cutter is a pleasure to guess the brutestones, where special accuracy is required, and anti-guardians or the tentacle of the best weapon just do not find.

    In the improvement scheme, seven empty nodes, that the standard, if compared with other weapons, but still quite a lot. First of all, it is necessary to improve damage, then the capacity of the closure, then speedflapeness and, finally, the speed of recharging. With monsters, six ammunition falls out, and in the clip in the backpack 25 is closed.

    Burian

    The bait slightly reminds the leaving, only more weighs and human hands do not grow out of the belly. Do not share the dimensions of the monster. It is not so awkward, as it may seem at first glance. Envie Isaac, he will seek forward, waving with huge blades. I do not think that the meeting will have to do you like. But the main danger of the monster is in his huge bouquency. Dozens of small monsters are sitting there. It is worth shooting into the stomach, how they instantly will be free.

    Having envy bryukatics, try to slow down it with a station. Then the shots cut the legs, and when he starts falling, gently shoot his head. According to the results, the monster is dead, and the belly is ko. If they still released monster freedom, switch to flamethrower or some other weapon beating in the area, and burn creatures. If one or two of the Waden hurry on Isaac, nothing terrible will happen. Quickly press the specified key to crush the enemy. But here if you attack the whole swarm, the hero will devourge alive.

    Laser rifle

    The laser rifle has such a wide radius of the lesion, which disappears the need for sure to dismember the monsters.

    Laser rifle on the principle of action resembles a plasma cutter - it shoots a wide ray of energy. Only this weapon has more lesion area, stronger shot, plus you can't rotate the trunk. Immediately talk about minuses. The first is low rapidity. It is much smaller, even with a plasma cutter. The second is a small clip, both at the weapons itself and in a backpack.

    The pros in the first place must be attributed to the monstrous power and the lesion area. Best weapon if you want to arrange minced monsters. My favorite reception is to slow down a few monsters, wait until the crowd is gathered in a narrow corridor, and shirt along the feet and hand. If there are improved damage, the units will survive. True, it automatically guarantees additional problems. Aiming shooting with this approach does not smell, so you have a weapon against Roy at hand.

    Alternative shooting range - Mina. Before rush, it is unwinding for some time, and then the deadly rays in all sides. A good attack against the crowd, but it is necessary that the monsters do not run away anywhere, so use the Stasis. If you choose this weapon, you prepare most of the backpack to give under ammunition. It fits too little in one clip.

    In the scheme, nine empty nodes, which is not at all cool. First of all, you need to invest in damage. Then, if there are problems with the place in the backpack, to improve the capacity of the clip. If there are no problems, then first in mine, and then in the clip. Then in the angle of attack. And finally, the last thing to be improved is the recharge rate. Two cartridges fall from monsters, six in the backpack holds six.

    Pulse rifle

    The pulse rifle is not too powerful, but it is convenient to shoot it from moving goals.

    Pulse rifle - ambiguous weapon. On the one hand, a small damage, and it is inconvenient to cut off limbs. But due to high rapidity, if you consider the damage caused by a second time, it turns out not bad. True, the higher the level of complexity, the less this plus. Monsters become a lively. In addition, a decent number of cartridges is closer to the clip, which greatly saves the place in the backpack.

    I used usually a pulsed rifle against small and shock creatures. If you miss - you do not have to spend time on recharging. And if you get, necromorph will stop and you can finish it. Also from the rifle, as it turned out, it is convenient to destroy bosses levels. You can almost instantly go from running to shoot and not worry if part of the charges will pass by the goal.

    Alternative attack - a circular shot. Isaac raises the weapon up and shoots 360 degrees. Not too useful because it is spent a lot of ammunition and suffer from a distance with accuracy. But if the enemy surrounded, you can use for defense. After speaking, you will certainly finish someone, thereby creating a corridor for flight.

    In the scheme, seven empty nodes. What to improve first? At first I would have improved the damage twice so that the monsters live worse, and then invested at the maximum in the capacity of the cloud. When 175 ammunition will be placed in the rifle, you will not only free the backpack, but also increase the efficiency in battle. Recharge speed can be left at last or do not do this improvement at all. 25 cartridges fall from the monsters, the service station is closed in a backpack.

    Guardians are usually hanging in front of the doors and do not let go. They look like a shapeless pile of meat with a human head and four tentacles sticking out of the chest. These monsters cannot walk, but it does not make them harmless. If you come to them closer, they will bring her head with one blow of the tentacles.

    When the guards notice the character, they begin to spit pieces from the belly. After a second, after the appearance of the pod, he grows the tentacle, from which he begins to shoot. You can kill the pods, cutting off the tentacle, but until you finish off the parent, they will constantly appear on the light. Guardians can be killed by cutting off all four tentacles. It may take time, because you have to aim at a distance. Therefore, it is desirable to slow down it with the Stasis so that the monster does not finish the pods.

    Flamethrower

    Industrial gas burner, perhaps, the most unsuccessful weapon in the game. It is convenient to burn small creatures, but, perhaps, that's all. Healthy monsters can be frying for a long time and stubbornly, and they will not experience special discomfort. Often for one necromorph and a whole climate is not enough. In addition, the flamethrower does not work in space. After improving the case, the damage is better, but useful return comes too late.

    Alternative attack - shot of a fire ball, which is exploded after a while. So he makes a bigger damage, but it is quite difficult to use. If you still decided to seriously use flamethrower, first of all improve damage. Then the burnt duration, then the capacity of the cylinder and the speed of recharging.

    In the scheme, eight empty nodes, which, in my opinion, is too much too much. The monster drops 25, khm, cartridges, and in total, 150 units are closed in a cylinder in a backpack. This is true, quite a bit at the speed of fuel consumption.

    Power pistol

    The power gun is not only damage, but also discards the monster.

    The same technologies are used in the power pistol as in the kinesis module. According to the enemy, a powerful power blow is applied, which breaks the bones. If the plasma cutter is a scalpel, the power gun is a huge dumina. An ordinary shot is not only monster cripples, but also throws back them back. Therefore, even if you do not finish off the necromorph, even throw away from myself.

    An alternative attack is a ball of energy that breaks through, and then spreads opponents. In general, a kind of pomegranate. The power gun is not accurate and not long-range weapons, it can even be compared with a shotgun. The best weapon acts against small enemies, as well as in 4 and 9 chapter, when the monster can be challenged into the zone of violated gravity and thereby instantly finish off, without spending extra cartridges.

    In the scheme, seven empty nodes. First, as usual, lift damage, then speed, then recharge (it is too slow) and the capacity of the clip. Three cartridges fall out of enemies, 15 in the clip climbs.

    Truborez

    The main source of ammunition in Dead Space - boarded monsters.

    Peeping with the flamethrower, I bought Truborez and realized that it was an excellent substitution with much more functional. Truboresis is also a melee weapon, so it is desirable, but not always be sure to slow down the opponent with the Stasis. Then press the shots key. The pipe cutting spits the circular blade, which rotates at a certain distance from the tool. While the blade did not stop, you can control it with flight.

    If the monster is controlled by the Stasis, slowly cut off the limbs to him and see how it is cute squeezed by pieces on the floor. If necromorp is active and hungry, quickly drive a saw on the legs to overlook, and then scold the remaining limbs. Similarly, small monsters are destroyed. They frozen them until they were sprayed at the corners, and chloride the saw until the necromorphic stops to twitch.

    Alternative attack - Shot Speed. She pierces all enemies on its path, but a small lesion area does not give a tangible result. How many I tried to adapt this attack for different situations, nothing sensible went out. Low speed-fire has reduced all the benefits obtained.

    In the scheme, eight empty nodes. Of course, we begin with damage, then we will certainly invest in the duration of sawing. In principle, this is enough, but it is still not bad to make a recharge and capacity (exactly in this order). 4 disks fall from monsters, in the clip of the backpack, 15.

    Power cutter

    Always follow the number of cartridges so as not to start recharge into such a bad moment.

    Power cutter, like a power gun, energy weapon, beating over the area. Only he has another principle of work. First you clamp the shot key and copy the energy. When it reaches the maximum, press the key to run the charge. One shot is usually enough to destroy several necromorphs.

    The snag, in fact, is that you need to save the charge. It turns out not always. The enemies do not like to stand and wait when you deign to the elbow. And not to use a crowd of monsters - useless waste of ammunition. Not only is four cartridges in the backpack, just four cartridges climbs in a backpack. Places are not enough disastrously, and after a serious dumping it will not be enough and ammunition.

    Alternative shot - Isaac directs the trunk down and shoots. Attack knocks down the enemy from the legs, at the same time apply damage. It works well against small monsters and when you are surrounded.

    Let's summarize. Yes, the power cutter is a deadly weapon, but to use it against single monsters - it's like shooting from guns on sparrows. Weapons effectively against the crowd. But even in this case, I preferred to use a laser rifle that does not require pre-charging before the shot.

    In the scheme, twelve empty nodes are an absolute record among the entire weapon. The first thing to be invested is damage. Then proceed to the charge, smoothly switch to the cloudy capacity and end the recharge rate. One cartridge falls from the enemy, four is fixed in the backpack.

    Passage

    The small repair ship "Kellion" is slowly approaching the Giant "Ishimur", which suddenly stopped contacting. It is impossible to say that this is a typical task. It is surprising that all means of communication failed on the ship of this class and scale. The extinguished overall lights cause even greater concerns.

    Docking begins smoothly, but suddenly the Kellion's shuttle dragges the gravitational field sharply, because of which the landing passes in extremely hard mode. The crew survived, but the ship was seriously injured, so to get out of the "Ishimuras", you will have to repair the engines. But while there is no speech about the flight, you first need to contact the "Ishimura" crew, which somehow do not rush to the landing platform.

    However, let me introduce you to actors. Isaac Clark - engineer and repairman, as well as the main character of the game. Zack Hammond - Security Service officer, Kender Daniels - Computer Systems Specialist. Two pilots we will not represent, they are prepared by the role of the first victims ...

    Arrival

    After landing, wait for you when you are returned, and go to the room on the stern to pick up the first-aid kit. Get out of the broken shuttle and go to comrades. Open the door and go to the waiting room. The elevator does not work, and you need to submit energy on it. Go to the room on the left, save the game and use the computer.

    The first date with the monsters we will bring to survive with bare hands. We'll have to see that there are strength.

    The elevator earned, but Monsters appeared, which quickly ate pilots. The beauty of the situation is that we have no weapons, so it remains only to run. It is impossible to get lost, the main thing is not to slow down the tempo. Reaching the elevator, quickly press the button to get away. When the lift stops, a hotel is a good room and pick up a plasma cutter from the table.

    Go to the closed door and destroy the panel. Now the pass is free. Concederate the monster, collect all the items and go to the transport system control room. Through the glass you will see Hammond and Kendra, who managed to escape. First of all, you need to establish the transport system to make it possible to move around the ship. Without it, it will not be possible to get out of the "Ishimura".

    Get out of the control room. Go ahead, carefully collecting all items. At the beginning of the game it is coming true. Seeing the Monster's corpse, palter to him on the legs to stop pretending. Reaching the large room, pick up the Stasis Module - Now you can slow down enemies and mechanisms. Go to the broken door, Palm with a Stasis on it and run into the corridor.

    Keep moving forward, collecting all items until you reach the repair shop. From the drawer on the wall, take the power unit and activate the left breathtaking mechanism. He clings to the transport trolley. Concederate the monsters and go to the second console. The exciting mechanism of the jacks, therefore, using the console, you need to immediately shove in the "Cogt" by the Stasis.

    When both breathtaking mechanisms can join the transport trolley, use the central console. Now the broken trolley is removed from the paths, but the repair is not finished yet. Therefore, disperse with monsters and go to the fought door. Look at the map where you need to move on. The previously blocked door is now open. Now you have to break through to the key that is needed to open the storage room.

    Despite the terrible appearance, in monsters, human traits are clearly guessed. Why it becomes more creepy.

    Finding the key, go to the storage room. I no longer say about monsters - they will delight throughout the path. In the storage room, take the power unit and upgrade your weapon or equipment on the layout. Take the data and exit the room. You need to return to the control room.

    You know the road, it remains only to break through the necromorphs. Download data into your computer and use the second remote to activate the transport system. Hammond and Kendra will go to the bridge, and you need to prepare Kellion to flight. Look at the card and go on the road. When you get into the waiting room, Hammond will contact you, which has safely reached the bridge. There is no one there, but computers still work.

    Well, just great, what else to say. Go to Kellion, go to the cockpit and use the computer. Something clearly went wrong because the ship began to explode rapidly. Run out of the ship and immediately take on the sight from the expectation room. From there, the crowd of monsters, who need to quickly shoot.

    After the battle, Kendra and Hammond will contact you. Now you have no shuttle, and therefore, the chances fall to get out of the "Ishimura" alive. However, we will not despair. While we are alive - there is hope. We need to find the Captain Body to get access codes. To do this, you need to get into the medical compartment. Hammond has already sent back the transport truck, so run faster to the station to complete the level.

    Small and shrinking creatures that are crawling through the walls and ceilings. Having freezing the hero, they occupy a convenient position, raise three tentacles and shoot. The charges fly along the broad arc, so dodged from the shot quite difficult. The sliders are easiest to kill when they are preparing for an attack. Wait until they subside the limbs, and gently shoot them. So that the monsters do not have time to attack, you can slow them down.

    You can try to shoot the sliders when they move, but because of high speed it is not so simple. Stasis also does not particularly help, because the charge of energy flies even slowly shot. There are two types of sliders - ordinary and improved. The second is characterized by skin color and health.

    Intensive therapy

    When the trolley stops, go to the wounded and dying doctor, which will give you the kinesis module. With the help of a kinesis, filming the trash at the door and go into the hall. Pick the flame down scheme and download it to the store. Now you can buy an industrial burner. Hammond will also contact you. He survived the attack of monsters, but lost the kendra. However, it does not change our plans - if we want to get out of the "Ishimuras", you need to pick up the access codes from the Captain body.

    There are absolutely safe places in the game. If you do not wait for the room, you can rush to you, even when you make purchases.

    The door to the morgue is littered with fragments, and to remove the challenge, we need special equipment. Hammond suggests that he found a thermal mix that can burn the hole. Therefore, make the necessary purchases, look at the map and go on the road. Slow the door to the Stasis and go to the laboratory. Clean the room from monsters, collect items and descend to the lower level. The security system will detect the presence of monsters and close the door. This is a signal you, so keep the weapon at the ready.

    Before entering the Kane office, slide the bookcase to find the cache. Then collect all items, write down the game and go to the biolaboratory. You will see a scientist who, like all the encirclances of the "Ishimura" crew, quickly blocks necromorf. Get acquainted with the new monster - the slider. Wait until the scientist will discard the skates, go into the room and shoot the creature.

    Collect objects and use the lift to get to the second level. Here you will find a small surprise, after which you can pick up a thermal mixture and other items that include a power unit. Return to the main room and the Stasis will slow down the door to slip, not afraid to be blown into two parts.

    Go to the break ... No, you can not use a thermal mixture, so by making buying in the store, look at the map again to understand where to go now. Once in the hall, raise the weapon to illuminate the road to the flashlight. Ahead of you will be waiting for the wonderful spectacle - the corpse, beating head of the wall. You can relax, now nothing happens. This is solely for the injection of the atmosphere.

    In the next room you need a kinesis to push the platform to get on the other side of the room. Let's bother with reclining monsters and assemble all items. Then a hotel name. There, too, a lot of interesting things are lying. Then return to the main room, pick up the power supply and insert it into the receiver in the wall to feed the lift.

    Tighten the platform to go to the other side, and finish off the monster. Walking the wall, drop the platform again to get to the door. Flip the control panel to open the door, and go inside. In the next room, the meteorite made a hole, and you will find yourself in open space. Therefore, follow the level of oxygen so that Isaac does not die from suffocation. But it is not necessary to hurry, the time and stock of the air is enough with an excess. Therefore, slowly wait all the lockers.

    Monsters from Roy are dangerous when pounces the crowd. Character burned instantly.

    In the following room there is no gravity, so get up with the thought that you can walk on the ceiling and walls, as well as jumping for dozens of meters ahead. Fly to the door, pick up a floating power supply and insert it into the receiver in the wall. Take the subject we need, collect money with cartridges and go back to the zero gravity zone. Two monsters are already swimming there, so learn to fight in new conditions. Now such contractions will occur quite often.

    Go out into space, finish off the necromorph and come back to the morgue. Now you have the necessary items, and you can burn the barricade. Return to the old way, just this time you will wait for small monsters. Break by the Stasis, and then burn out of the flamethrough. Having reached the barricades, install a bomb and wait until she clears the way. So we got into the medical compartment.

    Clean the room from monsters, collect items and insert the power supply to the receiver to open the door. Now it remains with the battles to break through the chamber of intensive therapy. Do not pay attention to the crazy doctor engaged in the patient, and immediately move on. On the way you are waiting for some unpleasant surprises, but in the end you will get to the morgue on the elevator. You will see how the captain turns into necromorph. Well, wait until we can do anything ...

    When the captain-necromorf and the contamizer with wild screams will be thrown on Isaac, push the monsters and collect objects. Be sure to take the access codes and the power unit. You have left to return to the transport station. On the way you will be sluggishly disturbing monsters. While you are writing along the corridor, Hammond will contact you and say that monsters are not the result of mutations, but an alien living form. But it is nonsense. It is much more worried about the refused engines. If we do not reopese them, the ship will simply collapse on the planet.

    Inmaptures

    These necromorphs look like flying skates. Their primary task is to create new monsters from corpses. Therefore, they usually do not pay attention to Aizek, preferring to create their feasures. Now, if you pay no one, then you will definitely attack.

    Inaptables need to be killed first until they have not had time to infect corpses. Miss this, and you will soon be provided with new monsters. These monsters are sufficiently weak and commemorate from the shot into the case. But for the best result, try to shoot the wings.

    Orbital correction

    Your global tasks at the level - run the centrifuge and fill the engines combustible. Collect all items and audio recording from which you will learn about the events on the ship. From the save point, go to the right and shine allegedly the dead monster. Sharply pretend to the robe.

    Kill the contamizers in the first place until they did from the corpses of new monsters.

    Go further, not forgetting to collect all items, at the same time visiting the store and workbench. Clean the corridor from monsters, and then kinesis lower the switch to activate the power. Go to the trolley and pull it with the kinesis. Sit down and use the console to move to the other side. There you are already meeting a delegation, so make a weapon. Having gathered trophies, find the power unit and climb the stairs to the second tier. Feature the pipeline and kinesis scheme. Activate the second remote.

    Shoot the many monsters, return to the trolley and go back. Now go to the dispatching. Through the glass you will see how the contamizer segments the corpses, so get ready for a meeting with necromorpses. Take advantage of the elevator to get to the next tier. Lockers, if you wish, improve the weapon and go to the disinfection room. Use the console to start the disinfection process.

    True, everything will not go according to plan. The light will go out in the room, and monsters climb through ventilation. Raise the weapon to turn on the flashlight, and shoot yourself from necromorphs. After beating the attack, you will unlock the doors and you can get out. Through the corridor - there is a gold chip and a power unit - you will get indoors with a centrifuge. Concederate the monsters so that you do not interfere with the task.

    In the room zero gravity, so boldly jump into the lower tier. You need to connect two large rotors to the centrifuge. But so that they successfully join, you need to pre-slow them in the Stasis. When both rotors are connected, climb the second level and use the console.

    The strength of gravity came to normal, but the air evaporated. We'll have to quickly get out of the room until the character suffocates from the lack of oxygen. Go down on the elevator to the first tier, and when the rotor will be used by you, quickly run to the next shelter. At the same time, do not forget about monsters. In one of the shelters you can recharge the oxygen stock. Reaching the elevator, go upstairs and leave the room. But this is not the end of adventure. When you find yourself in the corridor, Aizek will grab a huge tentacle and drag in Noura. Shoot the yellow bag to destroy the giant limb.

    To get rid of capture, shoot in a yellow bag.

    Translate the Spirit, look at the map where to go, and run to the elevator. Candra will contact you, which will begin to pump up Isaac with useful information. At the dispatching, visit the store, save the game and go to the fuel storage. You will have to go through the destroyed part of the ship in the open space. There is no direct path, so you have to jump on the debris. Just do not forget to assemble objects.

    Once upon a time, collect objects, among which there is a power unit, and go to the next tier on the elevator. Continue to move towards the goal, the fucked of small monsters. Go to the broken door. Wait until it cripples the necromorph, slow down it with a station and surpass inside. Braches Kanishy with fuel in small creatures, recharge all the weapons and go to the storage with fuel.

    Inside a full monsters, but also enough canister with flammable, so half of necromorphs can be killed, without spending the cartridges. By destroying monsters, recharge the weapon, restore health and use the remote. Now you need to reflect the second attack. Try to collect enemies into a bunch so that you can cover all of the laser rifle or throw a cylinder with fuel, if any remained. The battle will be difficult, so do not relax and, most importantly, do not stand in place.

    When you finish off all monsters, you will be released from the room. Go to the transport trolley. On the way, meet just one necromorp. Hammond will contact Aizek and say that they no longer threaten the fall on the planet. But then Kendra calls and report that the protection against asteroids does not work. This means that if you do nothing, the ship will soon be destroyed by stone blocks.

    Tentacle

    Sometimes the main character will be enough of a huge tentacle and drag behind him in Noura. It can not be slowed down, but you can destroy. When the tentacle stops to break, shoot in a yellow bag. You always have enough time to hit the monster.

    Death is inevitable

    It seems that Kendré concerns not a meteorite attack, but the fact that Hammond knows about Obelisk. While comrades are worn out, collect items and go to the cutting. You can not pay attention to the appearing and disappeared paw. With its owner, we will get to know a little later. On the bridge you will witness how the ship will hit the meteorite. Around everything climbs and starts collapsed. Quietly - this is not a reason for panic, without a script, nothing will fall to the end. Therefore, in a thoroughly awake boxes, save the game and go down.

    Freeze the creature by the Stasis and shoot in the back - there is simply no other vulnerable place.

    Pick up the third-level costume scheme. Now it is necessary at the first opportunity to attribute it to the store to immediately buy a new armor for 35,000 credits. While step into the management room to Hammond. He will say that nothing knows about Obelisk, and Kendra is extremely impressionable. And in any case, if Isaac does not repair the system of protection against asteroids, it will not become not relevant.

    Hammond can activate the system, but for this you need to redirect energy. And first you have to start the elevator. Go back to the bridge. When you come to the elevator, the door crashes the creature. Its armor can be poured only from the back, so first the stump in the back, go around and shoot until the monster is abolished. If we improved the weapon, it does not take much time.

    Pick up the trophy and go to the room where the monster jumped out. Native all lockers, activate meals and step to the elevator. Be sure to check the record while go down to the lower tier. Raise the power cutter diagram and wake the sleeping monster. Then, with a clean conscience, go to the side of the server room. Candra will contact you and says that there are some problems with gravity in the room. If you step on a piece of floor with a distorted field, Aizek spawn into pieces. This also applies to monsters.

    Gently bypassing broken plates, go ahead, knocking out monsters. If there is a power gun, you can throw their monsters into the fields with impaired gravity. True, the trophies will then have to select carefully. Recharge the Stasis and go to the server. There will be another creature on you. Turn the necromorph of the Stasis and shoot in the back, while it helplessly moves his paws.

    Select the power knot from the corpse and step into the corner of the room. Slow down the electrical cable when it is on the left, quickly run up to the console and turn off the power. This will redirect the energy in the first diagram. Return to the wheelhouse, finish off necromorphs and go to the elevator. You need to go down to the lowest level. While you are traveling on the lift, Kendra will contact you and tell that necromorphs appear by the transformation of dead fabric. Some monsters make corpses, others create new creatures.

    Upon arrival, look at the card to understand where to move on. It is worth passing forward as the partition collapses and you will find yourself in the open space. Faster run to the next room. Observe the monsters and, bypassing the violated gravity zones, go to the console. Redirect the energy in the second pattern and return to the elevator. Destroy the scratched necromorphs, go into the lift and go to the third level.

    You can temporarily stun a monster to be temporarily stunning, to escape or finish necromorph.

    Get out of the elevator and assemble things. Among other things, the hlama you will find a power pistol scheme. Prepare weapons and slowly move forward, getting ready to reflect the massive attack of monsters. Clearing the path, use the console to redirect the energy in the third scheme. Hammond will contact you and report that the protection system does not work yet, but the gun is activated and it can be managed manually. But to get to the gun, you will have to walk along the case.

    Drive to the next level, collect oxygen cylinders and leave out. The ship is periodically bombing asteroids, so run during the periods of the clutch, and the rest of the time is hitching for steel shelters. Having reached the cabin to the gun, select a ruby \u200b\u200bchip and a power unit and sit down in the chair. Your task: destroy asteroids while Hammond does not activate protection. At the beginning of the task protection of the hull 85%, if it falls to zero, Isaac will die. In general, the usual shooting range. Just do not overheat the gun, shoot short and accurate queues.

    When Hammond will turn on protection, you follow the global task at the level. Safety ship, but now something poisons air. You need to urgently eliminate the threat, otherwise you will simply suffocate from poisonous evaporations. Hammond is sent to the hydroponics section, Kender beats over the problem solving. You also have time to go. Run to the transport trolley. On the way there will be monsters, but it is nonsense.

    This necromorf is definitely dangerous, but with the right approach you will overcome it for one or two minutes. You always have to fight with creature. The creature loves to rush, pulling out thin walls and trying to crush Isaac. From afar, he can launch explosives, which, however, you can catch a kinesis and send back the owner.

    The creature is covered with impenetrable armor. You can maximum to cut off the legs, but necromorph does not particularly lose in speed. Therefore, wait until the monster surrenders on you, slow down the stasis and cut off the back. This is his vulnerable place. Shoot in the back of the most powerful weapons, not forgetting on time to immobilize the Stasis and bypassing a monster when it starts to spin. Having won the creature, you will get a power knot.

    Death Ranger

    Coming out of the trolleys, wait for the kendra. Piva faster spreads out in the air, and you need to hurry in the laboratory to get the ingredient for the antidote. Go to the security room, and with you will contact some religious fanatic, which will begin to broadcast about the natural selection and divine destination. Collect objects and keep in the direction of the diagnostic room.

    Do not forget to improve weapons, because monsters are gradually becoming healthier and healthier.

    Concederate the two slips and go to the elevator. Wall guards on the wall and climb the elevator to the second floor. Slow down the rolling platform and jump on the other side. If you do not have time to slip, Isaac is delimiting with electricity. Concederate the next slider, go around the wall, slow down the platform and run to the door. Go down on the elevator to the laboratory and go into the room to the right. Take the scheme and power unit and activate the console to get a reagent.

    When you take the reagent, the flaps on the window will open, and you can chat with a crazy doctor. The doctor will again begin to carry religious nonsense about the fall of mankind and necromorphs as a new and ideal life form. In general, everything is clear with him. But the doctor will decide not to limit themselves with one words and will challenge his brainchild - a hunter. This creature is almost immortal, so cut off your hands and legs, throw a stump into the stump to slow down the regeneration, and run out of the room.

    If Kender did not have time to unlock the castle, you will have to wait a bit. Ssed limbs and constantly freeze the stasis until you can get out of the room. Run into the diagnostic room. Monsters crawl there. Stop them from the Stasis and burn out of the flamethrough or crude pipe cutter. If it is a pity on the cartridges, you can throw a fuel cylinder that stands in the corner.

    Reaching to a safe place, pass and listen to the kendra. Now you need to get to the clinic to get DNA samples. Look at the map to understand where you need to go further. In a small room, you once again communicate with the doctor through the glass. Then you will go into a large room. Here you will meet a hunter with other necromorphs. Immediately calculate and freeze the hunter, and then take the rest of the monsters.

    When you overgo conventional necromorphs, the doors will automatically open. But if you want to collect things, do not forget to slow down the hunter in time. Once in the next corridor, you can force forget about the immortal monster. Collect objects, lift the canister and go along the corridor. Farm the cylinder on the left of the door and a highlight of the corridor on the right. At his end there is a crumpled nurse, which only acts on the nerves of an insane laugh. Pick up the items and move on.

    Go to the Office of the Svetable Doctor, collect items and use the computer. When DNA samples are ready, take the capsule and run into the hall. Our madman released the air, and now we need to quickly get to a safe place while there is something to breathe. In the corridor, slow down the stasis of small monsters and run past.

    All members of the Ishimura ship are already dead or, like this nurse, not a little.

    In a large room, where you met a hunter and a bunch of monsters, you will again meet the hunter and a bunch of monsters. It's not to fight with them at all with your hands, so the optimal option will cut off the leg hunter, slow down the other necromorphs and quickly run past. True, if there is no spare cylinder, you will have to replenish the reserve of oxygen from the device on the wall. It can be somewhat tightened.

    In any case, in the Security Room, use the console to seal the compartment. Now you need to do poison, so it's time to visit the chemical laboratory. While you are writing to the goal, Kender will report that, most likely, the air poisonously poison a huge creature of Leviafan, which populated in the compartment of hydroponics. Reaching the target, create poison, recharge the weapon and the Module of the Stasis and go to the Laboratory of Cryogenics.

    There you will meet the doctor again, who will reveal his plans. He wants to pick up necromorphs with him to the ground to attach to the Divine happiness of all the inhabitants of the planet. Oh, what a smart! True, while we do not get to the doctor, but now you will have to fight the hunter. This time no flight.

    First, by tradition, dismember and slow down the hunter. Then quickly destroy the sliders. Run into the control room, where the doctor stood, and wait for the hunter. Remember and immobinate again. Return to the main room and wait for the monster. Make him in the freezing section of exactly in the middle of the room. When he goes into the chamber, cut off the limb and barrier to the Stasis. Run to the control room and use the console to freeze the hunter forever.

    Now there are no problems. At least for a while. Take the power unit from the box on the wall and run to the transport compartment. Everything is ready to poison Leviafan and stop air pollution. It is time in the hydroponic compartment, where the poor fellow Hamond, probably, has already discovered from the nasty of necromorphs.

    These necromorphs with whom we will have to grab twice - peculiar bosses levels. They can not be killed by ordinary weapons. Even if you cut all the limbs, the monster quickly regenerates and rush back. It is necessary to kill with priests. In the meantime, you will not get to them, and this will only happen as it moves along the level, it will only be as followed. How best to neutralize the hunter? Cut his legs, then your hands, slow down the stasis and start to fall.

    Hazardous impurities

    Collect all items at the station and go to the elevator. Go down to the lower tier and find Hammond. He is still alive, but already barely breathe, so we need to hurry and clean the air faster. But comrade can still help the advice. He learned that the air spoil the killed and reversed staff of the hydroponics compartment. They need to be destroyed before leviafan come closely.

    Purchase the necessary items in the store and go to the western wing. Candra will contact you, which once again speaks his distrust of Hammond. Well, of course, he is a security officer who else to suspect? Go to the door on the left. In the corridor from the pipes, hot steam breaks out. In order not to burn, wait for a pause between emissions and quickly run a dangerous area.

    In the doors and narrow corridors it is convenient to shoot monsters. They have no space for maneuver.

    Carefully move forward, shooting monsters. Having reached the big room, recapture the weapon, become Mr. Caution and go quietly moving forward. From different sides, you will periodically jump off the growing necromorphs and try to break into shreds. Overcoming them all, a hotel's room and alternately visit small rooms on the right and left sides. There you will find one poisoner.

    Of course, when you finish off the poisoner and go to the hall, you will traditionally meet necromorphs. So, two creatures are ready, and you can move on. On the elevator, go up on the third floor. Pick up the items, go down to the second level and go to the room behind the elevator. Image Nicole will tell you that you are on the right track. Shoot the poisoner, quickly blame and throw a station in empty, until they drew the corpses.

    Return to the elevator and step into the room opposite. In the long corridor you will score two demolitioners. Shoot in an orange bag on your left hand until they come too close. Then you will go into a room with zero gravity. Jump to the hatch and kinesis open the lock. Concederate the monster and carefully not to get on the spikes jump on. We will have to suffer a little, but in the end you will achieve the goal.

    Go to the elevator and go down to the lower tier. Now you need to go through two corridors, which are periodically burned with jets of the flame. Not stepping into the first corridor, shoot the control panel to unlock the doors. When the flame falls, run to the middle corridor - here is a safe place. Bear from monsters, wait for the flame in the second corridor, and run through it. It remains to walk and finish the evidence. Now go back to the big hall, and from there step into the room, from where you started a trip to the western wing. Sell \u200b\u200bunnecessary items in the store and go to the eastern wing.

    Here you will welcome the guard. Cut off the tentacles until he finished monster. Native all rooms to collect a lot of useful items, and go to the big room. There is a creature on Izek. How to deal with it, you already know. Sourney from the Stasis, ride the back and shoot back from the most powerful weapons. Pick up the power unit and keep to the poisoner. You no longer attempt to stop.

    Concepting the living barrel with chemicals, go to the power control panel and press the button to raise the panel. When it opens, turn it over with the Stasis and look inside. Oh, another poisoner hid. Check off the infection and use the elevator in the center of the hall to climb the second floor. Collect everything that lies badly, go to the room and collect everything that lies badly.

    Yellow places are vulnerable points. This rule refers not only to tentacles, but to and other monsters.

    Further, Isaac grabbing tentacles and tries to drag in Noura. Shoot in an orange knot to put it plans. Freed from strong monster hugs, confidently go to zero gravity zone. See a faulty electrical device that breaks through the current on the floor? If you step on the panel, you will instantly die. Therefore, wait until the aggregate will stop the spokespring, throw the Stasis in it and quickly jump in the danger zone. Concederate the necromorphs. Do not forget to turn off the electricity, the remote is next to the device. Now go to the poisoner, shoot it and the old way Return to the corridor, where Isaac attacked the tentacle.

    Go to the elevator and go to the third floor. The last poisoner sits there. Before him, you will reach without incident, therefore, by eliminating the last source of toxins, come back to the large hall and break through into the center room. Use the mixture on the computer obtained at the fifth level to specify Leviafan. The poison will affect the monster, but will not kill. Well, you will have to deal with the old manner, with the help of guns. The battle will be held in the zone of zero gravity, so make the appropriate conclusions.

    Despite the impressive sizes, Leviathan is not too serious an opponent. At first he will piss the tentacles. Run from side to the side to dodge the attack. When the tentacles hit to the ground, you have a few seconds to fire the orange knot on the limb monster. So you have to destroy the three tentacles. The battle will go to the second stage.

    Leviathan will still attack tentacles, and much more active than the first time. But now they do not interest us. You need to blow into the mouth of yellow balls that play the role of shells. If leash, the monster will launch them in the main character. You can grab their kinesis and throw back, but it's easier to just jump to the side. Blowing up a certain number of balls in the grazing, you finish off Leviafan. This means that it is time to go to the transport compartment. Especially since Kendra came up with how to file a SOS signal.

    Discernant

    The demolitioner is a terrorist among necromorphs. His only goal is to dock to the main character and blew up. The explosion not only removes a lot of health, but also knocks down a character with legs. The demolition is not too harmful monster due to a small speed of movement. By and large, it is dangerous only in closed premises.

    Usually I managed to use a demolution against other monsters. The station is the monster and wait for the crowd next to him, the crowd is not formed. Make one shot, in a yellow bag on your left hand, and almost all opponents are roming at once. Beauty! Capital savings.

    Step into emptiness

    The kendra plan is quite simple in theory. You need to install a lighthouse on an asteroid and run a stone into space. With a lot of probability, he will fly away far away, and someone will hear a call for help. Coming out of the transport trolley, collect all items in the surrounding rooms - there are really a lot of them - and descend on the elevator into the processing room. When the lift starts the move, the monsters will jump on top. Therefore, stand in the corner and shoot a gun more powerful.

    Slide the legs to monsters to slow down the enemy. Crawling reptile is much easier to finish.

    First there is a fairly nervous walk along the corridors surrounded by necromorphs. Then, through a break in the wall on the left, you will reach the hangar for the processing of ore. You need to restore in the room the power of gravity to go on. But for this, you first need to remove all dangerous and unstable items. Grab small asteroids flying around the room, and throw them into a power beam. Double - after each successful throw - you will be disturbed by monsters, so do not lose caution.

    When all asteroids are destroyed, go to the console in the corner, but do not hurry to press the button. As soon as you restore gravity, there will be huge crowds of monsters on both sides. Therefore, it makes sense at the cost of the power block to hack the room behind the console and keep defense there. Necromorphs will jump off the ceiling, but one by one, and you can easily kill them. And the prizes found in the room will allow not only to buy a new power unit, but even stay in a small plus.

    Then go to the unlocked door. Select the access key, collect other items and come back to the elevator. Just do not forget to sell in the store any trash. You will not get to the point. Then go to the elevator and go to the lowest level. This time no one will try to devour.

    Coming out of the elevator, quickly finish off the contamizers until they finished monsters. Then a kinesis tighten the trolley and go on it to the other side. On the right and the left will pop up the monsters and shoot through Aisku any fat. If you will quickly destroy necromorphs, you will have time to even collect all prizes, scattered on the side platforms. Reaching the point, you will see Nicole on the other side. It turns out that it is still not a hallucination? She will open the door to you, but you need to cover the girl.

    Necromorphs will mostly try to get Nicole, but somewhat will attack Aizek. In any case, you must think first of all about the girl. She is enough for several blows to go to heaven. By completing the task, you will get access to the room, but Nicole will run away. Well, we can not change anything. Go to the room, pick a lighthouse, a fourth-level costume scheme and other useful things. Then return to the trolley back. Monsters will be pounce on you again, so keep the weapon at the ready. Just in the elevator - the contamizers again in the case - and go to the second tier.

    Sometimes monsters in the literal sense jumped on Isaac. To remove the enemy, click on the key specified on the screen.

    In the room where you will find yourself on arrival, nothing really can be seen and full of monsters. Therefore, wear them to the elevator, where much better with visibility and more conveniently aim. Go to the room to the left - there is a store, you can buy armor, - remove the power supply from the receiver, throw it back to the dusty room and finish off the monsters. Reap the power supply and carry it into the room opposite. Insert the block into the receiver to activate the elevator. Now you can get to the asteroid.

    Crashing with monsters, carefully inspect the hangar. You need to destroy four rotors that hold Kamenyuk. The first is on the floor, the second on the ceiling. Wherever, you can understand the power beam, going from an asteroid. You are in the zero gravity zone, so it's not a problem to get to the Rotor. Slow down the mechanism of the Stasis and through the hole in the side protection shoot the power supply.

    The remaining two rotors are on the other side of the asteroid, and you will have to get into the trim. Wait until the clips go out, jump onto a stone and run out quickly. Skip necromorphs, break the rotors with an old way and put the lighthouse on the asteroid. This needs to be done quickly - the oxygen reserves are rapidly falling, and we still need to have time to come back inside the ship.

    Return. The room with dust is now embraced by a flame, but remained safe places, and you can slip up to the elevator (small, in the room on the right). True, you first need to return the power supply to the receiver, where did you get it from. It is not too difficult, the main thing is not to hurry and throw the block forward using the kinesis.

    On the elevator, go to the control room, collect items and use the console to send a lighthouse with a stone in a free flight. It remains to hope that someone will receive a signal. However, even if it is so, we will not know the answer. Antenna is broken, and it needs to be fixed. Return to the transport station through a large elevator. You will attack necromorphs - there are many of them, so do not lose caution.

    Find and save

    Here we were already, so you should easily navigate the terrain. Run to the bridge, not forgetting to pre-visit the store. On the bridge, the electronic security service will notify the danger, closes all the inputs and outputs and leave you alone with monsters. From the ceiling will begin to actively jump out different creatures, among which there will be many demolitions. Given that necromorphs can in the literal sense to fall on the head, do not stand in place and constantly move.

    After beating the attack, you will remove quarantine. We run into the room, where the creature comes off, and take the power block from the drawer on the wall. Collect objects left after the battle, and go to the elevator. Drive on the third tier. Candra will contact you and tell about the current situation. In the next room there are two guards, which is not a joke at all. Prepare for this meeting with maximum seriousness and caution. There is a small trick - until you drone over the corner, the pods of the second monster can not shoot you.

    Periodically, you will meet comrades, but, unfortunately, but these meetings are short and not too frequent. Most of the time you have to spend alone.

    In the next corridor, you are waiting for one of the most opposite necromorphs - spruit. Fix his legs and kill the creatures on the back. So that they do not crawl, do not forget to throw the Stasis. On the body of the spruit you will find a ruby \u200b\u200bchip. Earn debris kinesis near the entrance and go to the elevator. Reaching the next tier, select the power unit and go through the tunnel on the transport trolley. This time no one will attack you. You will fall into the room of communication, from where you can go to the antenna itself.

    Collect all items in the control room and store to the antenna. Standing around the entrance, finish off the jumping necromorphs. Now carefully inspect the device. Half batteries does not work, but we have and remaining. Your task is to remove non-working batteries and put the workers in such a way that a circle formed around the center. In the room zero gravity, so collecting devices - a matter of two minutes. After that, the antenna will earn, and you can take a signal.

    The warship received the SOS signal and heads to Ishimur. True, he still picked up a rescue capsule, which was one of the creatures that Hammond was blocked. It would be necessary to warn the military, and then the confusion may turn out. But the transmitting antenna also does not work. More precisely, it works, but something big prevents it from transmitting a signal. You need to quickly get rid of the monster and contact the vessel.

    Get out of the room and go to the elevator on the trolley. In it, by the way, a monster will sit, so keep the weapon at the ready. On the elevator go down to the lower tier and run to the gun control room. Collect objects, among them the power unit, and sit down in the arrow chair. You need to destroy the monster that entered the antenna. You will have to shoot four tentacles. You need to shoot, as usual, in yellow bags.

    But the monster does not intend to just give up. It will flush in you pieces of insulating "Ishimura" and every garbage. Armor at the beginning of the battle is 100%. If she falls to zero, Aizek will die. Shells can be destroyed on the nettle, but it is not so easy. You will throw away all four limbs, try you to keep track of everyone! It is better to focus on the destruction of the supreets, as far as possible trying to shoot down the wreckage.

    When you destroy the monster, the kendra will start passing the signal, but it will be too late. You will see how the captain of the ship finishes necromorph. Then the military vessel will dock in the "Ishimuru" and Isaac will lose consciousness. When the character finds, Hammond will contact you and says that his signal has blocked. It is very strange, but there is a good news. Looks like Comrade found a working shuttle.

    To start the ship, you need to get to the victim wreck warship and drag one important item. The dark future again lit up the light of hope, and you need to hurry to the transport station. On the way you will meet several necromorphs, including maliciously sprouts. The last monster, it will be a bruky, in general will wait for the transport.

    This lanky monster is one of the most opposite necromorphs in the game. It moves not too quickly, but it can hit the tentacle from a decent distance or generally cling to the character. It is not difficult to shoot it, but the death of the monster is only the beginning. After death from the back of the necromorf, small spruits are separated, which are quickly crawling and hurts to the head of Aizek. They are even more dangerous than Roy Blukiatics.

    Try to shoot the boards from afar. When the monsters start from the back, slow down them with a station and carry, until they squandered around the room. Or slow down the spruit, cut down his legs and, while it falls on the floor, burn the monster from flamethrough. True, if it flames without improvement, a lot of fuel will go to kill spruits.

    Dead upon arrival

    Upon arrival, Hammond will contact you and say that a warship turned out to be nearby. He purposefully flew to the "Ishimuru", but not for a rescue operation, but to destroy the ship. This is why it does not seem amazing. Go to the arms room of a broken ship. During the crash, there was a leak of radioactive elements, and until you cleanse the room from them, it will not work further.

    In zero gravity areas, you can jump for huge distances.

    This is zone zero gravity. Get down to the big hatch and destroy six mechanisms on the sides. In order to save ammunition, you can beat the foot. Luke will open, and then all air will disappear. Now look carefully for the amount of oxygen. Shoot the sliders and start the kinesis to collect and throw out the green spheres in the open hatch. Periodically, you will be disturbed by necromorphs.

    Cleaning the room, jump to the exit and leave the zone of weightlessness. On the ship itself everything broke and explodes. Therefore, you will constantly stumble on the zones with impaired gravity. Passing a little forward, you will see the running monster. Do not shoot him after it will not return anyway. In the cargo compartment, kinesis move the containers to go on. Soon you will meet with new necromorpses - runners. Try to slow down their stasis or cut down your legs to stop the enemy.

    Turn left and disassemble the block before the door to go further. Continue moving forward, shooting monsters. Try to open the door that leads to the upper deck. You will not succeed, but the necromorphs will appear from behind. Concederate the monsters, select the power supply and insert it into the receiver. This activates the elevator on which you will reach the upper deck.

    On the way to the upper deck, Dr. Kane will contact you, which will say that the flight is not the best option of action in the current situation. You need to deliver the obelisk on the planet to stop it madness, otherwise humanity is doomed. Go to the weapon warehouse and shoot the spruit with its spinal friendships. Observe other monsters, collect objects (there is one power unit) and leave the warehouse.

    However, if you do not rush anywhere, you can participate in the competition for accuracy and shooting speed. To do this, go to the shooter to the left and use the remote. You need to shoot red shapes and do not touch blue. You do not have the right to error. For the first and second victory you will receive ammunition, for the third - average first aid kit, for the fourth - ruby \u200b\u200bchip, for the fifth - power unit.

    On the way to the medical compartment you will meet a deadly wounded soldier. To pass the next room, you need to freeze the laser installation and carefully slip by the deadly rays. Just before this wait until all curious monsters perish, and assemble the scattered objects. Run forward, shooting monsters until you reach the barracks (just do not miss the power unit).

    Burkytikov need to be killed especially carefully not to release small monsters from the nest.

    Come inside and go ahead a bit. The script will work, and monsters attack you. There will be a lot of them before indecency. Constantly move and do not spare the cartridges, the battle is really complicated. Okole Nekromorfs, start collecting trophies. In addition to the objects dropping out of monsters, in the hall is full of boxes, plus a lot of interesting things in the side niches. One good - all unnecessary can be sold in the store, which is here.

    Having reached the machine compartment, climb up the elevator. You need to turn off the engines, and for this you have to cut out the power. The problem is that the room periodically perfectly helps. Carefully look at the incomprehensible barrel-shaped devices on the left and right. You can stand behind them, and you will not hit the flame. These devices will serve for our protection. Become for the object and cling to its kinesis. Take a little sideway, then push forward.

    Look at the center of the room carefully. See Purple devices? They need to be destroyed by taught shots. In total, they are three pieces on each side. By destroying everything, you turn off the flame and you can pick up the kernel. That's just it will become the last blow for the ship, which will start rapidly fall apart. Run where the navigator indicates until Hammond meets. This is your last meeting, because the comrade is flooding the creature. Wait until the monster shakes the glass, turn it over with a station and shoot in the back.

    Take a diamond chip from the corpse and run out faster from the engine room. Remove the power supply near the elevator and put it in the old receiver to block the door. Now run to the weapon room, where we get rid of radioactive elements, fly to the exit and keep to the transport station. On the way, monsters will be thrown on you, but this is already a tradition that does not even surprise.

    Runners - the result of turning the corpses of soldiers equipped with a stasis module in necromorphs. These monsters are strong, fast and twig. They are desirable to slow down the Stasis to not be particularly running. But it turns out not always, because they move too tightly. Therefore, better keep the powerful gun at the ready to shoot down the runners on the approach.

    The last days

    One kernel for the launch of the shuttle was not enough. We need to find three navigation cards. But to get into the room where they are stored, you need to get an access key. Go to the living room, where the crew members committed mass suicide, and, by completing economic affairs, go through the door at the end of the hall. In the corridor through the glass you will see how the crazy doctor finishes the surviving crew member and leave the room. Go to the dining room, where murder was made, and go on the elevator to the lower tier.

    Below you will be waiting for two boards that need to quickly neutralize. Native a room, take the power unit and the access key and on the elevator, go back to the dining room. Dr. Kane will contact you and ask you to meet him when you find three navigation cards. Previously, he will not open the doors. Get out of the dining room and go to residential premises. Concederate the necromorphs and open the key the door leading to block A.

    As soon as you step in the bedroom, Izeka grab the tentacles and drag in Noura. Shoot in an orange knot to get rid. Kinesis Open the magnetic lock, collect all the items and go to the Block B. In it failed the climate control system, and the bedroom turned into a huge freezer. And without oxygen. Disappear with monsters, replenish air reserves and go to the next room. Reconnect the necromorphs again, select the navigation map and go back to the hall where the door opened the key.

    In the Hall you will attack demolitions. Having coped with them, go to the block S. Go to the basketball court. In the locker rooms, select a scheme for level 5 armor. In the basketball hall, it is zone zero gravity - jump onto the platform ahead and select a navigation map. In the room immediately materialized the sliders. Do not stand in place, jump from the site to the site so that you do not go from the rear.

    If you want to get prizes, do not rush to leave the pad. You can play cosmic basketball. The rules are simple enough. The kinesis is enough flying ball, jumping on a litior area and throw the ball into the ring. You are limited to general time and time on a specific throw. The prizes are. The first and second round - ammunition, the third is the middle aid kit, the fourth - loans, the fifth - ruby \u200b\u200bchip.

    Get out from the basketball court and store into the block C. In the room, where the crazy doctor is broadcasting from the monitors screens about the need for self-sacrifice, finishing the monsters and go to the bedroom. You need to move the bed with the help of a crucible in such a way that you can pass into the opposite part of the room. Just remember what and where to move back, because soon you will have to flee.

    Visit the last navigation map, turn it out immediately. A crazy doctor again put the hunter on you. True, incomprehensible, old or new. Immediately shoot limbs and slow down the monster from the Stasis. Quickly stop the bed back to get out of the room and run to the exit. The door, as discharged, is closed, and while Kender does not break the castle, you must keep defense.

    Soon obelisk, monster and in general the whole colony will come to an end. They will fall a piece of planet size with a small continent.

    You will have to shoot from ordinary necromorphs and hunter. The latter need to constantly dismember and slow down the stasis, so as not to be confused under the legs. When Kendra opens the castle, run to the room, where the crew members commit suicide. Do not forget to slow down the stasis and dismemberous monsters.

    Reaching the room with corpses, quickly find and destroy two emissions. If you miss them, they will raise all the dead and have to deal with the monsters ord. Cleaning the room, sell items in the store and go to Dr. Kane. He will open the door and tell that all the monsters telepathically controls the main monster who lives on the planet. The only way to stop him is to return the obelisk on the planet.

    You need to get to shuttle. You will go through the cabins of officers who need to be united for the presence of valuable items. Reaching shuttle, collect all items, go to the ship and download navigation cards. Nothing bad did not happen that it surprises slightly. Go to the control room and use the console to activate the system.

    You did, but the hunter appeared behind the back. Now the full order. Slow it and run out of the dispatch. Other necromorphs appeared on the moisture deck. Obschery yourself with them, and then join the hunter to the shuttle engines. Dismember, slow down the stasis and run to the dispatching. Use the second remote to start the engines. The hunter grieves and no longer disturb you.

    It remains to release clips and see how Dr. Kane sits into the ship. Now you need to deliver an obelisk to the ship. Get out of the control room, select the power knot from the bodies of the hunter. Run to the transport station. You will again be disturbed by monsters, but much more interesting is that the crazy doctor will contact Iizek, who will again pronounce the sermon and bring himself sacrificing to the contashinger. What enthusiast ...

    Alternative solution

    After reaching the desired station, run in the cargo compartment. Monsters a little. Having reached the remote control in the cargo compartment, take items from the boxes and the power block from the container on the wall and press the button to raise the obelisk from the depth of the vessel.

    Pushing the trolley at the end of the game is a clear bust. It will have to drag as much as two levels.

    You need to push the trolley with obelisk to the cargo compartment. This is done so. Grab the carts of the kinesis and drag on the rails. As soon as you start moving, the crowd of necromorphs is immediately materialized. Also, the tentacles will be raised from the depths of the ship, which needs to be destroyed by shooting orange bags. After pulling the trolley to the lift, which will move the obelisk to the hangar, go back to the transport station.

    From there, run to the hangar, exterminating the monsters. The loading system does not work, so you have to load the obelisk manually. Run into the dispatching and turn off the gravity. Now go down to the hangar, finish off necromorphs and run to the obelisk. You need to bring the trolley to the ship's trim. Twice with a computer will have to translate the arrow on the rails to drag the cart. And of course, necromorphs will be rushed at you.

    Return to Dispatcher and turn on gravity. Now run to the ship to meet Kane. True, it's not destiny. Doctor will shoot kendra! She, it turns out, works for the government and must deliver Obelisk to Earth. Moreover, Kendra will tell that Obelisk was created by people. This is an unsuccessful copy of the alien artifact, which was found on the ground. Here the theory of alien mind failed. All that happens only the consequences of an unsuccessful experiment.

    Nicole's conversation intervenes in the conversation, which will call Aizek to the Flight Management Center. Run there - the guard on the wall, a lot of things in the lockers - and listen to what the beloved of the main character will say. Nicole came up with how to stop the kendra, and soon the shuttle will begin to return to the "Ishimuru". True, the traitor was able to escape in a rescue capsule. But it is no longer important. Run to the ship. It's time to return the damned artifact on the planet.

    Jumpers do not have legs and move exclusively to Clear. However, they do not suffer from this feature. You will see for yourself when you see, at what speed they are crawling on the ceilings and walls. The jumper can simply underwent and hit the tail, but it does not happen to attack and jump. Especially they like to throw out this trick in zero gravity areas.

    Jumpers move quickly enough, so try to slow them down with the Stasis. To hit the monster, you need to shoot your arms and often head. If you do not want to spend the Stasis, wait until the monster crashes almost close, and shoot hands. If you get a jumper to stop, and you can finish it.

    Dead space

    Exit the shuttle and tighten the loader to the cargo compartment. Stand Obelisk, as far as you can, on the rails and go to the room on the left. Open the room on the right and quickly finish off the contamines until they have time to get home and turn a bunch of corpses in monsters. Then Name all the boxes, select the power unit and pick up the power supply. Get to exit, throw it on the ground and go to the street.

    But Kendre said that you should not try to take the artifact from the planet ...

    Shoot the boards on the street, go back behind the power supply and insert it into the receiver to apply for the database gate. Go to any open door - the rooms are opposite each other and lead in one place - and shook the obelisk inside. Use the console to open the second gate inside the base.

    Come to the warehouse. You should take a step forward, as a crowd of monsters will appear. Run around the room, shooting from monsters until they all die. It is necessary to push the obelisk to the next throughput. This is done like this. Use the console to raise the bridge. While he did not let go, freeze the bridge of the Stasis and quickly carry a trolley forward. Enemies will periodically pop up from ventilation as the obelisk is promoted. From one of the necromorphs even falls out the ruby \u200b\u200bchip.

    Open and close the gate to get into the next room. Go to the door on the right. There are two guards that need to quickly cry. The control panels of the bridges do not work, so run on the door where monsters hung. You will fall into a long tube with zero gravity. Run up, shooting oncoming mandes, until you get to the top. Use the console to give meals.

    Together with the bridges, fans earned, so the walk back will be more difficult. You need not only shoot monsters, but also slip through the fans. So that you do not quarrel into pieces, overclap them before jumping. In the room you will attack the creature. Act the old scheme. First, the stasis is slowed down, then go around behind and shoot back. You will no longer find resistance, so you can safely push the obelisk to the site of the excavation. Just do not forget to collect all items in the rooms.

    Continue to pull Obelisk. On the left of the abyss will get out three tentacles. Shoot on orange bags to destroy them. Then the crowd of monsters will be scored on the base of you. Keep the defense until necromorphs are completed. Then collect trophies, push the obelisk to the very edge and activate the lift. Go to the room to the right and wait until disinfection end. Perhaps we rushed with care, because Kendra appeared and, pushing speech, was lucky to the clan.

    When the monster grab the Isaac by the leg, continue to shoot in the eyes. And do not break, otherwise go straight to the mouth.

    It is urgent to catch it up. Run along the corridor. If they reached the room with a point of saving, shop, workbench and a bunch of objects, it means that the final battle awaits you. Go outside and see how a huge monster will finish off Kendra, and then, turning the retreat to the tentacle, will take it for Isaque. However, it is not so terrible, like his little, and with the main monster you will handle no problem.

    Monster attacks tentacles, just like Leviafan. You need to run away to avoid shocks, and shoot in yellow eyes around the mouth. When you destroy three eyes, the monster grab the Isaac behind the leg and moves down the head. This is not a reason for panic, so continue to shoot from this position along the two remaining eyes. Having deprived a monster of vision - although the eyes did it be? - You will find yourself on Earth.

    The monster scratchfully squeezed and starts more actively by the tentacles. However, not so to stop us. Now you need to destroy five yellow bags in the chest-grazing. It's harder to get on them, so wait until the monster bends closer. Also I advise you to refuse to throw cylinders with flammable. All the same tentacles will knock them out before you have time to make a throw.

    By destroying yellow bags, you finish off the monster and release the way to shuttle. Faster run into the ship and enjoy not fully understandable, but the final roller.



    Wait for the captions, and make a record. Now you can download it to start the game first, but with all weapons and improvements that have done as Dead Space passing. You will also receive 50,000 credits, 10 power blocks and prize armor, which can be bought at any store for 99,000 credits.

    Introduction: (several rules for a pleasant pass)

    • When you meet with a monster, do not waste the cartridges, it will be more effective to shoot them limbs, i.e. Hands, legs, head. How did you understand the monster killed? Shoot all limbs and it will be 100% corpse =)
    • Do not forget to collect resources and objects, they will be needed for pumping ammunition and weapons.
    • Some of the huge monsters have weaknesses, they are usually highlighted and painted in yellow-orange color (color ocher). Just gently aim there and shoot him all ^_^
    • Oh, I almost forgot, you will probably be wondering where we are stored. So, the retainers in this game look in the form of boxes that resemble a camera hanging on the wall.
    • From myself I will say that the first best weapon - plasma cutterbut still to kill a large number of meat running on you will fit plasma rifle.

    In the game 12 chapters:

    • Chapter One: Arrival
    • Chapter Second: Intensive Therapy
    • Chapter Three: Orbital Correction
    • Chapter Fourth: Death is inevitable
    • Chapter Five: Death Addiction
    • Chapter Six: Hazardous impurities
    • Chapter Seventh: Step into Void
    • Chapter Eighth: Find and save
    • Ninth chapter: dead upon arrival
    • Chapter Ten: Last Days
    • Chapter Eleventh: Alternative Solutions
    • Chapter Twelfth: Dead Space

    Little preface.

    2507 year. Resources on Earth are completely exhausted. For survival, humanity remains only one way - space, searching for resources on other planets. In the course of cosmos studies, the found planets become an ideal source of minerals. Huge cargo ships - the leaving replete planet behind the planet. One of the ships - Ishimura did not touch ...

    A group of specialists at Shuttle Callion immediately sends to find out the reason for the disappearance of communication.

    Well, we start the game!

    Chapter 1: Arrival

    Mission Ses ER529

    Status….

    USG "Kellion", course - anegis system ...

    The main composition of the crew consists of 5 people

    • Izek Clark - Main Hero - Engineer (Technician), Ship Systems Specialist
    • Kender Daniels - Computer Systems Specialist
    • Zach Hemond - Senior Security Service officer
    • Caprals Chen and Johnston - Pilots

    DETEKTIVE A: Play the location of the mining ship of the USG "Ishimura" ...

    Testing in: Install and eliminate the cause of communication problems ...

    Our protagonist (GG) Isaac is sitting sniffing, listening to the discussion of the team located on Shuttle Callion. It never happened to be connected with Ishimuria. After landing, just follow your team and follow their requests. You will be asked to open the door, for this go to the door and click " E." (default). If you are lost, click " IN"She will tell you the way. Next you will need to activate the console. Expect a cute scene) Your task is to run! And as soon as possible, there is nothing faster. Your salvation is an elevator. Here you met with monsters - necromorpses.

    Necromorf - (Greek. Morphe - the form, NECRO - death) - an alien form of life, most often found in the form of mutated corpses. Roughly speaking, bacterium necrocofers falls into the deceased organism and instantly begins to multiply, leading to mutation of the entire body. As a result, the corpses come to life and change the shape of the body - fangs, blades, tail, etc. appear. The organs are also undergoing mutations. Want to kill necromorph, shoot them limbs.

    This is how necromorph looks like:

    When descend down on the elevator, get into the room with a corpse, near it eloquent inscription "Cut Their Limbs" - "Cut them limbs". As I said above, it is better to do this. On the table you will find your first weapon (congratulations!) - Flame cutter. Perhaps the best weapon for instant cut-off limbs. Swim corpses and drawers, you will find a lot of useful in them.

    The flame cutter has damage 10-18, its ammunition is 10-20 rounds, recharging time 1-0.5 sec. Figure 1-0.6 sec. per shot. It also has an alternative mode (vertical). This is a rotation of 90 degrees. Suitable for improvement on the workbench

    Passing in front, you will see Kendra and Zack, they are the only ones who could survive. You will be asked to fix the transport system. On the way, do not forget to preserve. For the future - not all corpses - corpses ) Therefore, if we noticed a strange body on the floor, it is better to shoot at it from afar, it will greatly facilitate life. Otherwise you will, like me, bounce on the chair, having heard the beautiful voice of necromorf with your back ^^ Do not skip the resources that are hidden in boxes. Because In the future, you can improve your things on the workbench. The first obstacle is the jammed door, raise the stasis module, a very useful thing, aim at the door and click " FROM"And we can go through, just do not delay, because Stasis slows objects for a while.

    Stasis The module has a charge level to be replenished. With the help of the Stasis, you can slow down not only the doors, but still objects and opponents that you strongly do, use it when you do not have time to kill monsters running on you. Stasis will help move some details in the game. Suitable for improvement on the workbench

    You get into the room where the mechanism needs to be launched. We kill everyone. Go forward.

    In one of the rooms you will find on the wall of the power knot, in the boxes - ammunition, on the table - the information panel. Also in the room there is a workbench.

    The workbench is a complex of instruments to improve the parameters of the GG. With it, you can improve all weapons that you have with you, your costume - X or Stasis module. For improvement, power nodes are needed. And you can lay out extra things. (He works as a warehouse.)

    We return. Good-quality portion of monsters on the way you are provided \u003d R Next We use "B" and go ahead to the adventure. On the way, we deal with necromorpses. After overclocking the demonstration of necromorphs, go to the landing terminal, and then to the landing station, on the way exploring the restroom and breaking the boxes. We go to the cargo elevator who quickly takes you down. To the right of you will be the store.

    The store is the only way in Dead Space to buy, and not find ammunition, aid kits, weapons, power nodes. In addition, in the store you can buy new costumes that will become available as the game is passing.

    Putting on all the money with shaking hands, go to the trolley. She will bring you to the next chapter.

    Chapter 2: Intensive therapy

    At the outlet of the trolleys, we see a woman, do not be afraid, approach, she will give you a useful device - kinesis module.

    Kinetic pistol - allows you to move items at a distance, including taking objects and fold them into monsters. Suitable for improvement on the workbench.

    For the passage further use the resulting device. Go forward. You will get into the room with three doors. One of them is leading to the intensive care unit, barricaded, will have to blow it up. The components of explosives - glycerin and the electric industry you have to find in the remaining two doors. Native all rooms and move forward. It is not necessary to scare greatly when the light turns off and starts to squeak siren, just accept the position of more comfortable, you have a battle not with one monster, I advise you better. We are moving forward with the fight (do not forget to use the button " IN"). You can sigh when you hear that quarantine is removed. We explore all the premises. We get into the office, we take the video there, search all, the cabinets can be moved to the kinesis, so you will get access to the secret room, where you will find a gold semiconductor - a very big value that can be sold in the store.

    On the first floor you will meet a new kind of monster, which will be destined to kill. Next on the elevator rose above. It will be the main laboratory, here you can find explosives and power knot. Semore everything, come back. Do not forget to recharge your gun \u003d R. On the way, we persist and shoot it, the game calls you into the diagnostic wing.

    (Scene) - I'll go on the wall ... This phrase is wonderful for our corpse.

    We move forward to the elevator. It is de-energized, but this is not a problem, we use the power supply located nearby. It must be inserted into the connector on the wall with a zipper sign. Before entering the elevator, inspect all the rooms. Next, boldly put in the elevator and climb to the second level. Rising, you will get into a large room with a mechanism. Your task is to move this mechanism using a kinosis. Get ready to shoot. After disassembly, you will again have to move the mechanism (platform), now climb the elevator and go further. Fun! Fun! Fun! \u003d R.

    Here you will find a walk in space, but do not be afraid - the air reserve in your scaffle will not let you die, but oxygen can end, so you should not consider raven. In such places there are usually oxygen stations. The next door, and now you are already in the usual corridors. Next is an interesting and huge hall with a door at the opposite end and a huge abyss in the middle. The only way to move is to turn off gravity, good, the remote is near. Now, jumping, you can gain a foothold on any surface - the magnetic shoes of your suit will provide a reliable clutch. You just will tell you about it. We feed the door, as they did before, we move, collecting everything along the way forward. We picked up the electric hand. We are great, but coming out of the room, we need to defend them again. We repeat manipulation with the block. By the way, here you will get to know the new type of monster - larvae (Swarmer). They are easier to kill fire, if they jumped on you, press " E."But they bite pain. Move forward to a large room, here you can change the costume or fasten. Bring the barrier. Zack will say that Isaque needs to go to the morgue. We go to the morgue, killing all the non-living. You probably wanted to find out how these monsters get? One of the ways - a monster, resembling a bat, docked his own trunk to the brain and infects the body by bacteria, which transform the corpse, reviving him. All this you will see when you get into a large room with a glazed office. It is desirable to kill the resulting kind of monster as soon as possible, it is sufficiently smart and strong. If you do not have time, do not spare the stasis. And also the advice - kill this mouse first, otherwise it makes many necromorphs. We return. We are in the engine room, the path to which again runs through the transport deck.

    Chapter 3: Orbital Correction

    Arriving at the next place of events, you will again get instructions:

    • It is necessary to restore the gravity on the ship;
    • It is necessary to restore the supply of fuel into engines.

    You will have to kill several necromorphs, after which in a small room on the first floor you can find an ammunition with a scheme, there is also a workbench. Slash. In one place (use the locator - " IN") You can drag the switch down, while activating the refueling mode. After waging on dark corridors and already hung up to the fog somewhere near a wandering monster, you can find a power knot. We move forward, persist. And finally, we find the second lever. You report that fuel is served. Now you need to go back, but the path will be not easy. A large number of mutants awaits you. Further your path leads to a centrifuge where you can get to the elevator through another dispatching door. Before you enter the room, it is thoroughly a good wardrobes and pick up the ammunition. In the room, by clicking on the button, you will run the disinfection process, it will attract the attention of a huge number of mutants that will constantly appear in the room using the ventilation pipe. To cope, use Stasis and torn off limbs of enemies. Ahead of you is waiting for zone zero gravity with monsters, which are not averse to lower your health. We understand with them. To restore the supply of energy, it is necessary to freeze large rotating shafts with the help of a station and immediately insert them into the central part of the centrifuge. Turn on the power centrifuge from the console at the top: the gravity will return, and the whole garbage will be sent to open space. We move forward. And here we build a house of bricks. Who is what is much. Be prepared for the attack of a giant tentacles that grab you. To get rid of capture, shoot yellow-orange thighs on it. According to the corridors, we run back, killing necromorphs. We need to go to the engines. Moving in the direction of the locator. It will lead us to a large room, where we activate the engines. You will find a massive attack of monsters, be alert. Get together again to the computer to confirm the ignition. Now that the engines are running and the orbit is adjusted, you will be waiting for a new problem - the ship moves into the asteroid field, which can destroy the ship, and the defense is deactivated. Isaque need to go to the trolley. Hamond will wait for him on the bridge.

    Chapter 4: Death is inevitable

    When you meet with Haymond, from his words, repair the management of weapons of opposing defense in his power, but for this you need the energy for which the three key nodes are responsible, which is the Isoque and have to be repaired. But before you need to activate the elevators through the security room. You will certainly continue the passage of the game Dead Space and go to the open door, but I advise you first of all to search the bridge for useful things. You need to return to the big hall where the meteorite fell. Here you will find a meeting with a new, very dangerous type of monsters - bruta.

    Brut. - This is a huge necromorph covered with a thick layer of armor (reminder crab shell), which is impenetrable for ordinary weapons. The only way to win it is to shoot on the shoulders, or rather - in the yellowighs on them. The ideal option is to freeze with the help of a station and slowly, with taste, to separated a carcass. By the way, do not forget to search it then.

    We use the Stasis often, you have a slight advantage - the monster moves more slowly. You still have a tip from me personally - if you manage to get around it, then in the back on his back there are already familiar yellow-orange growths, aiming in them, you will kill him much faster. Native all rooms. Move further along the corridor, destroying everything along the way and thoroughly hitting up the blue rays - they are deadly dangerous. Observe the brutal again. There are two electrical cables in front of you, under which I do not advise you. To pass them, you need to use the Stasis. There you will find the first node.

    To get to it and turn it on, you need to freeze cables. Return to the elevator from which they came, and come back to the top (if you forgot the way, then use the locator). This time in the hall you have to deal with a considerable number of opponents. Take advantage of another elevator in the center. At the outlet of the elevator you will find an audio log and schema. On the walls you will find an ammunition and a point of preservation, and a little further - a couple of strong opponents. Behind them - node 2.

    After that go in the elevator. When you find yourself in a big hall, you are preparing to shoot. Carefully examine the hall, and also go to all the doors. In addition, there is a door closed with a power lock (you have to spend the power knot), there is quite a lot of ammunition behind it. Continue to move right around the hall, you will find the third node in the end.

    Well, that's ready-made oscillator weapons to shoot, you only need to repair the information cable to target the shooting on asteroids. We use the locator, you will have to return back, rising by several elevators and driving in the monster hall. You need to get on the ship's hull outside. Zack gives a good advice - look for shelter not to be killed asteroids. Do not forget that the reserve of air is limited. You can still move through jumps between objects at a high distance. Going to the gateway chamber, on the left, take the semiconductor, can also be preserved. Your starry hour !!! Sit into the chair, check your accuracy and reaction =) After several misses, and maybe mats xdd. The route will turn on, and you can go back to continue passing.

    This time there will be no meteorites on your path, it will be several necromorphs that will try to kill you. It would seem, you will finally be able to breathe calmly, but it was not there. Soon you can not breathe at all. Something poisons air and your next task is to find the necessary medical preparations to stop the creature. Hammond is sent to the hydroponics department to fight the creature, and in the meantime you need to find funds to fight. You need to repeat the whole path back and return to the transport station. After killing a couple of monsters and coming to the trolley, you will end the passage of this stage.

    You can rest \u003d p

    Chapter 5: Death Prestilation

    The passage of chapter begins with a session with Kendra. According to her, the infected air extends faster than she counted. The only hope survive is to prepare a chemical weapon for the essence of air. And you need to do it quickly. You can find the necessary ingredients in a chemical laboratory. You were held next to her in the second chapter, but this time Kendra crashes the door leading to the laboratory for you. Thank you very much.

    On the running paths, go to the room with three doors. There you come to the second door to the right "Imaging Diagnostics Wing", judging by the voices that are distributed in the room, you are not the only surviving ship. Further on the long corridor. He will lead you to the diagnostic room, where you will meet a new kind of monster. To go on It will be necessary to take advantage of the Stasis. Next we go into a chimlaborator. The creatures in the tanks unambiguously hint that someone experimented with the original material of necromorphs, combining it with human, and in addition to the will of the last. In the center of the room there is a device for creating chemical mixtures, use it, To get a chemical capsule ( E.to take). Suddenly the glass will open, and you will see a crazy doctor who decided that the days of mankind have long expired and it's time to move on. The result of such a "movement" suddenly will live in one of the chambers. It is useless to fight him - you just can't kill it, cut limbs will be recovered. The only way is to escape, pre-frozen the creature by the Stasis and cutting off his legs. Fight up from the room, the creature will remain there, and you have to come back with small necromorpses. Go to the clinic not to get lost, use the card or locator, you will again be a meeting with the doctor. Get out of the room, get to the big hall where hell awaits you ( xdd.): In addition to the regenerating necromorf, you have to destroy even a huge number of creatures. Actively use the Stasis and try to delay the immortal necrophore until Kendra opens the door lock. (Here is the asshole, you think) and a newly dark corridor. At the end of it on the wall there will be another creature, to the left of it, in the corner - semiconductor. Finally, you will reach a small room forced by cans with tissue samples, near the audio magazine, on the floor ammunition and semiconductor. In the room you will find the device, download the chemical capsule into it, and after a few seconds you will receive a practically ready-made chemical, but it should again be attributed to the chemical laboratory. In the meantime, it is necessary to continue the passage of the game Dead Space and get out quickly from the compartment - an abnormal doctor "dumped air", and you need to return back, making your way through large groups of necromorphs. In order not to get lost, use the card and locator. The path is not enough, but the air is small, so that you move. Reaching the room with three doors, get to the system responsible for air supply, turn it on. Now your path again lies in a chimlaborator. On joys, I will inform you - the path will be free there, in the device mix the ingredients and get the final chemical weapon. Now it is necessary to return to the transport station. A mad doctor will be released. He will tell you about his plan. A new door will open - you go there, on the wall - the point of preservation, in a large row of cabinets - ammunition. In the big room, you will learn that this psychopath wants to transfer infection to Earth, but until you stop it - it's time to deal with his immortal necromorph. It's just impossible to kill it, but freeze forever - quite. Your task is to lure him into the freezing station. The challenge is to run beyond the legs and freeze monster by the Stasis. Run to the room at the top, there is a cryosal control center there, press the key on the console and watch your almost invincible enemy goes to a long freeze, go there and the road. Native a room. You can exhale, the second door leads you to almost the transport station.

    Chapter 6: Hazardous impurities

    At the station, raise the audio recording, it will give you the initial information about what is happening in the hydroponics department. Moving forward, rising on the elevator, you will meet with barely Hamond. He will tell you that it was able to know that there is a huge creature in the food storage. It is it that is the root cause of air pollution. Workers of the deck of hydroponics mutated, turning into air pollutants. If you want to survive - you have to kill them all, more precisely - eight, but only then destroy a huge creature - Leviathan.

    Leviathan is a lot of creature. The body is circular, there is a similarity of the mouth, three tentacles. Yellow mines fly out from mouth, which explode when hit, they can be shot down at a distance. (Excellent weapons against Leviathan well proven a pulsed rifle and a plasma cutter).

    You will get into a room with several doors. One of them leading to the food storage is still closed, the other leads to the hydroponics department, where there are infected workers.

    Following the card prompts or locator, move on. Try not to fall under the evaporation of the gas. We go ahead, we go on the elevator, and we get into the "Western Greenhouse". From now on, be extremely attentive and careful, use the locator.

    You get in the huge size of the room that pays many dangers, monsters and supplies. The locator will lead you to the room with the contamizer in the middle, the right sign is greenish air and the use of the oxygen cylinder. It is not necessary to shoot in a creature, it's good enough to get through it.

    Again, a large hall and a similar room in another end, the finale with the contagious is the same.

    On the elevator, which is located in the middle of the hall, raise the third, the top floor, you can find a semiconductor and a little ammunition. It is better to do it before visiting the second floor, since then the road will not return here. Go down to the second level. The locator will lead you again into an infected room, at the end of which the third contamin will sit on the left. However, as soon as you kill him, a surprise awaits you. There will be a couple of creatures that can revive the corpses and turn them into necromorphs.

    Quickly kill them until they have created an army against you. Your task is to get to the room to another

    side of the hall. Be careful, here you have to face the fuse.

    If you shoot in a yellow thigh on your hand, it will explode, so it's not worth doing this when the opponent is close. You can cut off your hand with a bomb and thwart it in other opponents. Hell continues. You will get into a large room with a lack of gravity. To continue Promotion, you need to remove the damper from the ceiling using a crucible. It will be quite a few necromorphs. Having finished with them, go straight to the opening you open. We use the locator, moving forward, do not forget to interact with the switches.

    When you get into the room with pipes, be extremely careful, I hope you should not say that it is not necessary to go into the pipe when the fire is soaked in it. Tip: Disable the switches (second and third tubes), while in the doorway. Use the locator and move forward. As soon as you go through all the cutting pipes, go into a small room, you will find the next contagious.

    We are moving around the locator to the second greenhouse. I make my way forward by the corridor, there are waiting for mutants there. If you can use the power knot and open the door, tick the taste. We run to the elevator, we go to the level above. You will face Bruta, and a little further there will be another contagious.

    The next contagious hid from us among oxygen cylinders.

    Immediately it is necessary to activate the cylinder replacement system. On the elevator we go to the third level. There we are awaiting the penultimate contagious. Use the locator and move towards the latest contagious.

    screenshots: (left to right)





    When you meet with a tentacle, remember my advice =) Shoot him in the thief. We run along the corridor, the locator leads us to zero zero gravity - carefully! We blocked the road drum with electrical energy to pass, just use the Stasis.

    Close up, there in the corner you will find the last contagious. To promote, you need to restore the oxygen supply. We return. The way back will be practically free. Locator will hold you to the console, confirm the activation of the air recirculation system. The door leading to the food storage will be available. Insert the flask with chemicals in the device, the door ahead will open.

    Just a friendly Leviathan.

    Do not be afraid, come closer, just collect all the supplies first ^^

    Liviaphanchik wants to stroke you with his tentacles (for the scenes - what is the charm of Tentakley :R)

    This is one of his attacks, the second - he will spit in you by Minami. It will not be difficult to cope with it if your vigilance and attention will not let you down. Shoot in mines and growths. With Minami Liviafan, you can play Pinpong: we catch them and throw them into it. Fun!! Try to move more often, because Blowing sucks are quite painful.

    After victory, return to the transport deck. The kendra appeared a rescue plan. You must send the SOS signal. Let be free and clean, calmly walk around the ship.

    Chapter 7: Step into Void

    This chapter begins with connection with Kendra. The situation is sad, you need to find a radio beacon that can transmit a signal to the rescue ship, attach it to asteroid \u003d R.

    This asteroid was specially prepared for smelting. You have to send it to open space to obtain a high-quality and strong signal.

    The logging is closed, but do not be discouraged, somewhere here, wading on this chapter in our favorite game, you will find a spare key. (Just like in Pinocchio, everything is going to work) Well, we went to look for this key, I hope you are ready ) Go…

    You are right on the corridor until you reach the doors. Left will be a power node. Over the door will, no, no, do not be afraid, it's just an impressive sizes of a corridor in which you will find a lot of useful for yourself, look good. At the end of this corridor there will be an elevator. The elevator can deliver you to four decks: 1 ( A.),2(B.),3(C.),4(D.). You yourself are on the first - A., the game leads you to the second deck - IN. I advise you not to lower the gun, you will have to fight. Because Monsters will jump on Isaac from the roof. Going to the deck, go straight and left - the next series of monsters attacks will be waiting for you. Use " IN»For landmark. Near the save point should turn left and go down. Break in the wall will lead you to zone zero gravity. According to the kendra, you need to restore gravity. But for this, it is necessary to throw meteorites that are in the room (4 pieces, the amount sensor is on the wall) straight into the flame that buys indoors.

    To do this, drag meteorites with Kinesesa. Be prepared to bring off the attack of necromorphs, there will be quite a lot of them in the hall. When destroying meteorites, gravity will appear, and with it and new opponents. Next to the gravity activation panel you will find a room that requires a power node. Getting inside, you will find a little ammunition and semiconductor. In the control room on the opposite side you will find the access key - it will come in handy you further in the passage of Dead Space, this is the same key.

    Go to the second door - it will be faster, to the right on the wall there will be a power node. Soon you will reach the elevator leading to other decks. With it, go down to the deck - D.. Be vigilant - you are waiting for a pair of necromorphs that are able to revive the corpses. It is advisable to figure them out quickly. Native career carefully (in the corner you will find a gold semiconductor). Go to the door, tighten the trolley using a kinosis. As soon as you press the start button, you immediately look carefully on the sides - there will be necromorphs that can shoot in you. Since their number is great, but there is nowhere to run, then you need to deal with them quickly.

    When I coming to the end, Izeka will begin with glitches - he will see Nicole. She will help to open the room. Protect Nicole.

    Use the kinesis to tighten the supplies to yourself. Take all the supplies in the room and the lighthouse. In addition, the room on the rack is a fourth-level protective suit scheme - it can help you well.

    We return. Shoot and not scared =) On the elevator we are going to the deck - FROM.

    Your task is to turn off the gravitational clamps that hold the asteroid.

    We use the locator, moving forward, cutting off from necromorphs. By the corridor you can get to the elevator, but first look at the left - here you will find a workbench and shop. You need to take the battery and attribute it to the opposite end of the level. There will be a de-energized nest, where the battery needs to insert. To clarify the location, use the card. By inserting the battery into the nest, go down on the elevator nearby. Be careful, you will attack demolitions. A long corridor will lead you to considerable ammunition reserves and a gold semiconductor.

    When you get into a huge room with zero gravity, you will need to turn off four disks. Two of them are in the hall, two - outside. You can get out, having jumped on the asteroid, and devoid of the vacuum to him, do not forget to install an asteroid on the top of the asteroid (in a specially reserved place) of the radio transmitter. To stop the disk, you need to get up on the side and shoot into the inner part, it hides the metal rotating plate, it can be frozen. All this time you will be born necromorphs. Next, use a card or locator, go back, just carefully. We reach the elevator and rise above. We reach the door of the cabinet mining management cabinet, we use the found key. Do not forget to collect all the supplies. Activate the console. Now the signal is broadcast. In addition, there is another problem - the transmitter works, but with the problem receiver. You need to go to the bridge and, if possible, fix it. But it would seem that the safe way back to the transport module is suddenly broken down - a quarantine is turned on in the office, and the monsters begged from the ventilation holes. We place them all in porridge)

    Have fun, go to the deck - BUT. Be on a check, in the elevator will be unsafe. Using the locator, return to your favorite transport deck.

    Chapter 8: Find and save

    URAA! You were heard, a warship flies to save you. The only problem is that they do not hear you. Kendra asks you to fix it. You need to get to the bridge and from there to the Center for Communication. Do not forget to buy in stores and use the workbench. There will be quite a few necromorphs on your way, especially rush into the black eyes, they are stronger. We understand with them and go to the elevator, we are on the third level. Passing a little further along the corridor, you will find a new kind of opponents - Divider. (divided), it is easy to learn on high height and hoodoobye, after death this creature will fall into a lot of small, each of which strives you to kill, to fight with them weapons of mass lesion.

    Moving forward using " IN", Return to trolley, run it. We go ahead on the task. Through big round doors you will reach zone zero gravity. Pay attention to the wall - see a schematic representation of working (green) and non-working (red) transmitters? You need to rearrange the transmitters with the help of a kinosis so that in the first circle there are only working transmitters. That is, the technology is the following - from the first circle kinesis throw off the non-working (broken, marked) transmitters and instead of them all the same kinesis set the workers (with blue backlight). Elementary =)

    Returning to the panel, send our coordinates. By activating the panel, you will learn that the ship that was supposed to save you took the rescue boat with necromorph. I think you should not explain what now can happen on board the rescue ship? Perhaps you have a chance to warn them. But the signal is very weak - something organic and unequivocally huge closed the entire communication zone outside the ship. Drive on the trolley back. Instead of an open elevator, you will see a black necromorph, coping with him, quietly descend down. On the cargo elevator rise up. Now sit down for the turret.

    How are you a carcass? ))

    Shoot the time in the monster so that it reacted. Tentacles will appear with yellow-orange growths, on them and need to shoot, only beware of the debris and barrels flying in you, they will spoil your ship. On the right there is an indicator that shows how many percent of the protective shield remains, do not allow it to fall to zero if you do not want to start passing from the next point of saving. When you kill the creature, Kendra will try to contact the rescue ship, on the video on the screen we see that it is too late. The ship crashes into the Ishimuru, Isaac loses consciousness. Finally, Hamond leaves, someone, according to him, blocked the signal. He discovered a working shuttle on which you can escape from the Ishimura. To check this, you will have to go to the cargo compartment. On the elevator, go down, then along the corridor to the second elevator, again downward, will find yourself on the deck, here you will find another dividing opponent. Repeat the path back to the vehicle compartment.

    Chapter 9: dead upon arrival

    The head begins with a dialogue with Hamond, he will make you one news. That rescue ship - Weilor, It didn't happen at all. The ship was equipped for a combat operation. Unfortunately, it is not possible to find out everything to the end yet ... again there is no interference. After with us, Kendra contact us, she will tell that he again lost him (she meant the connection with Hamond) we need to find a converter and dumping from here. Go ahead. You enter the zone of weightlessness, you need to throw out flying green swabs into open space.

    But first of all go down to the bottom and destroy the Luke Locks. Get ready to shoot, and even replenish oxygen as needed. Do not forget to collect resources. When you cope with radioactive materials, locks are unlocked to not be lost, use the locator. Kender will contact Aizek again. She will tell you what you need to find. The task is to find the necessary parts in the engine compartment, on the stern. Straight, naturally, you will not get, you will have to go around. Well, we go. Once in the room, we fear Shuschikov (Tip: When attacking with them, use the Stasis if problems arise) I think that you will master the way, your task is to move the boxes with a kinesis, I advise you to save every corner. Oh yeah, if you reincarnate when killing these shuschikov, I propose to remove stress on the corpses =) may fall out xdd. Okay, when you try away from the soul, go ahead. Save, the light will go out, and two necromorphs will appear in the room capable of reviving the corpses. Do not need to make your dirty business. After you need to power the battery elevator. While you will go to the elevator, the survivor will contact you - Dr. Terrence Kane. Indeed, Vaylor's crew knew much more than you, arriving at the damn Ishimur. The marker is the source of all that is happening until he returns to the planet, from where he was taken, hordes of necromorphs will threaten all mankind. The shuttle, which you may be able to repair, should be used to return the marker on the planet, only then this incessant nightmare will be over. Go around the challenge, turning right, a little right there is a small room with supplies, at the end of the corridor door. You will find a shared monster, explosives and a power knot on the left wall. Nearby will be a tire.

    Here you can have fun.

    We persistently recommend that you take your skills in it. And this is why: there are 5 levels in the dash, for each passing level you will receive a prize. First level - cartridges, fourth - semiconductor, fifth - power node. For victory, you need to shoot in red figures, blue - do not touch. Make everything quickly and clearly and significantly improve your position. Having written in plenty, it's time to return to passage =)

    We are moving forward and find the very martyr ... Immediately follow him into the room. I DO NOT ADVISE get under lasers. Using the locator, moving forward, hold the gun at the ready. Kill in the next 3-5 minutes have a lot ^^

    After passing a long corridor with an impressive number of necromorphs, soon you will reach the engineering compartment. Collect supplies. In the middle of the corridor you will be treated right, on the open elevator, climb upstairs. You will fall into the Hall of hello flame Bugaga ^^ So, your task will be a kinesis push a huge engine O_O. Or something similar to him, he will protect you from fire, provided if you stand as close as possible to it. Side the switches that need to be shooting, otherwise you are torture Aizek (Tip: If you still roasted the poor fellow, Izek will become unmanaged, at such a moment, so as not to die, you can use the first-key first aid kit, sometimes it saves. These actions will have to do From both sides. Covering with this task, we take the converter. Now your path lies before the deck of the crew.

    We observe an epic scene ...

    i will not spoil \u003d R. ah, well, here is your screenshot:

    Tip: This bruta is about x2 HP, but its shoulders and torso are now protected much better, consider it with his mini-babe of this chapter =) Well, in general, you somehow kill him there, and I'll write a continuation further (he-hehe).

    How to kill the annoying monster, do not miss the eye of the diamond conductor. Use the locator, run to the elevator, fuel it by inserting the battery. Quickly run away from Waylor. Kill monster. Here we are at the transport station.

    Chapter 10: Recent Days

    For you there are two news - good and bad. A good - Kender knows where the shuttle and in general is ready to run it, bad - someone removed navigation cards from the shuttle. Now you have to find them on the deck. In this chapter, I advise you to use the locator more often.

    When you get to the elevator and climb to the top floor, you will be expected a cisss of, where a mad doctor kills the surviving as a sacrifice. Unfortunately, before the doctor you can not get there, the entrance to this room is a little further along the corridor, on the right. In the room you will find some supplies and on the elevator can go down.

    A little further, in the corner, near a woman who took the wall, in the box you will find what is looking for. Be careful - the woman is not completely dead. To continue passing, come back to the room where the murder occurred, and from there in the corridor. Move forward using the locator.

    Ahead of you is waiting for a lot of battles \u003d) fighting off the enemies, you will get into the room with gravity and many monsters. In addition to necromorphs, there are cylinders to replenish oxygen. Use the card to have enough air. Native adjacent rooms. Pass on the corridor and find a navigation map.

    When you get to the elevator and go down, you will fall back into narrow corridors, also on the way to the right you can find a scheme. I draw your attention to this scheme, as it will allow you to buy a protective suit of the fifth level in the store. Next on Earth you will find a text message with a detailed description of the new mini game - Z-Ball - We will tell you a little later. In the meantime, open the door and get into an unusual gaming room, where there is no gravity. Before you start the game, select the navigation map in the middle of the hall and disperse with necromorpses. The essence of the game is quite simple. You need to jump on the ignition platforms when they touch them, they will go out, after tapping the last platform, catch the likeness of the basketball bowl and throw it into the hole. And so several times. Time limited. The game has five levels. For each passing level provided a prize.

    After the game, using the locator, go ahead, killing everyone in your way))

    Reaching the elevator, which will be in the room, go down to the big hall. Here you will find a good fight with monsters. Explore all the rooms in the hall - you will find a lot of interesting things, including personal audio recording of the captain. Next we go into large doors in the middle. You can get straight to the shuttle, set the navigation cards, visit the next room and try running the shuttle engines.

    You will again be a meeting with an immortal necromorph. It's time to end with this. Make the creature to the shuttle engines, shoot your legs, freeze the stasis and run into the room nearby. Run engines check. And make a kebab from it)

    You will again see Dr. Kane, he will sit in the shuttle and ask you to let go of the clips of the shuttle - what to do. After that, you remain the last passage of the tenth chapter. In place of the immortal monster grilled, select the power unit, explore the deck and assemble the ammunition. Everything, you can now return back (use the card and locator), by the way, killing the necromorf returning to the life and his new sacrifice is a crazy doctor. Return to the transport station.

    Chapter 11: Alternative Solutions

    Welcome to the penultimate chapter of the game!

    Dr. Kane will contact you to explain the essence of the task - it is necessary to find a cargo compartment, in it to find the marker and with the help of a cargo elevator to deliver to the hangar. In the transport compartment you will find a workbench and shop. To make it easier to make the passage at this stage, use the locator and the card, however, it is easy to find the road and without using them. Soon you will get into a large cargo compartment, having broken with necromorpses, reach the console and activate it, after a few seconds, the marker will appear. You can move it using a crucible, but you will be very disturbed, guess who? ) That's right - necromorphs, and then - tentacles for tentacles. In the transfers between the attacks on the transport ribbon, pull the marker to the elevator on the opposite side. As soon as the marker turns out to be on the platform, it will be raised into the planting compartment, where you need to move and you. The lack of a marker is clearly not failed to taste with numerous local inhabitants, so the way back will be hard to call a light walk.

    Climb the level above the elevator, then through the doors to the corridor and the transport station. Here Kendra will restore the supply of elevator energy, and you can get into it (the new door will be available at the transport station). Eat, do not rush, check every corner ;) Then return to the corridor and go to the big door. Let's get into the room with a point of saving, here you will find an unpleasant surprise in the form of a sudden quarantine and attack of opponents. Return to the room that was blocked on quarantine, and pass through the large door, you will find yourself in the hangar. Soon Dr. Kane will fly to the shuttle.

    The automatic loading system does not work, so you have to turn off the gravity and load the marker manually. Using the locator, get to the control room. There you need to drag the marker with the help of a cruise. When you get to the arrow, switch it on the panel, boil the marker to perpendicular rails, switch the arrow again and lead it further to the shuttle.

    You will completely interfere with necromorphs. In the second arrow you will need to pull the marker right under the shuttle, then you need to restore gravity. According to the corridors, get to the shuttle. You do not have enough of only a few seconds - Dr. Kane will fall from the shot, and the traitor, I will not spare, hits the shuttle. The next communication session will also clarify, including the origin of the Marker.

    And Kendra will be a government agent, which only it was necessary that the marker.

    You were thrown to die here. Or not?! Unexpectedly, Nicole will be released - your beloved, the one for which you arrived. Glitches? Isaque straight road to the flight control room. As we have already got used to running back, killing everything in our path, and maybe something else will pick up that they missed first (I am about resources). When you find Nicole, she will tell you the "pleasant" news ... You can control the shuttle remotely. Kendra will have time to erase in the rescue boat. Leave the control room. Go to the landing platform.

    Chapter 12: Dead Space

    This last chapter begins on the surface of the planet. Get out of the ship, Nicole will tell you the use of the loader to get a marker from the shuttle. Carefully look around, there is a small mobile platform nearby - the loader, it must be tightened with the help of a kinosis to the ship. You can pull the platform only to the boards of the ship, there will be a control panel - it needs to be activated, the obelisk will automatically plunge and, now with the help of a kinesis, you will need to pull the obelisk along with the loader.

    Tip - Some bridges will have to freeze.

    To go on, you need to find a battery. We go into the only open doors, if you need, we use the workbench and the store, we go further and kill a couple of monsters, take the battery and taking it to the closed door. Patch the marker. Here there will be fun and the smell of gunpowder vita ....

    It will have to run a lot, just use the locator. Make a task (turn on the fan) cool run on huge ventilation right? =) We return back and still pull the marker further. After another buffer room, you will find yourself on the street. Landscape is simply unreal.

    Take the obelisk to the edge. When you drag the obelisk further, surprise awaits you ^^ Sea Action =) You not only need to give a marker, but also to fight back from tentacles and necromorphs =) Well, I hope you coped with this task. Now Isaque needs to make legs from this planet. By the way, there will be another surprise in the form of kendra O_O. She's back. And while you will be in the room, you can watch a complete message from Nicole. Unfortunately, your beloved is no longer there, and the fact that you have seen it was only the fruit of imagination, supported by the mystical forces of the Marker.

    Very funny fact - if you fold the first letters of the name of the heads of Dead Space in English, we will get the phrase: Nicole Is Dead - Nicole Dead

    Use the locator and get to the shuttle, a small scene (I advise you to pogly and turn off the light ^^ You will see the death of the kendra and immediately meet the main boss of the whole game, and now catch how much the aid kits do you have? And how many cartridges? ))) And the name of it Mind of Ulya.

    Bogagangagaga

    Okay, I believe in you ;)

    The mind of the hive is the main boss game, the head of all necromorphs. He did not leave the planet, but telepathically controlled all necromorphs. It is this creature guilty in all misfortunes in the game, and you will have to kill it if you want to finish the passage of Dead Space. Mind of the hive represents more worm-like creature. Around the mouth, if it can be called that, yellowighs are located. The same growths are located in the chest, but they first closed the similarity of the ribs. To destroy the creature, you must first destroy the yellowighs around the mouth, and then, when the edges "open", - yellow thighs on the chest.

    Weapons can choose any, but I advise you to take something smart, for example Rifle.If the cartridges on it are a bit, here are the alternatives: plasma cutter, power cutter, pipeline. It should be attacked when the creature will "sweep" on you, approaching the maximum distance.

    Do not be afraid when she grab you, continue to play in the insane Max !! =) Shoot and do not stop for a second - this is the entire deposit of victory. Naturally, the tentacles should be destroyed. Also, exploding canisters who carefully left by developers can also use damage to yellowighting. Be prepared for the fact that the mind will spill a couple of necromorphs - you will also need to cope with them. Then the attack follows using exploding balls, dine. When the last thigh will be blown up, you will finally see how the boss died, because of which your favorite died. You revenged.

    EVERYTHING! You are great, and now run to the ship. Here is the end of a great game Dead Space. .

    .
    2. Intensive therapy (Intensive Care).
    3. Orbital Correction (Correction).
    4. Death is inevitable (Obliteration Imminent).
    5. Lethal Devotion.
    6. Hazardous impurities (Environmental Hazard).
    7. Step into emptiness (INTO THE VOID).
    8. Find and save (Search and Rescue).
    .
    .
    .
    .

  • Questions - Answers .
  • Walkthrough

    9. Dead on arrival (Dead ON Arrival)
    Dead Space How to go. Chapter 9.

    Officer Hammond proposes to pick up spare parts from the Wailor warship of the warship and repair the local shuttle. Then the connection disappears, communicate with Kendra about the same.


    Ore Storage

    We go to the zone of weightlessness. At the bottom of the huge flaps we hit 6 fuses with legs, they are located on both sides. After that, the gate will open, the air will begin to be consumed. We need to catch flying round rose balls, and throw them into space. One of them is hidden on the wall behind the round platform. Near the open flaps are two panels, where you can replenish the oxygen stock. After throwing all the balls, we enter into a deepening in the wall, get on board the Veilor ship.


    Cargo Bay.

    A very fast necromorph will run away from us in the corridors. We go to the site where the kinesis needs to move magnetic boxes to pass forward. In lateral deadlocks on the right and on the left you can find cartridges, if you move unnecessary boxes. After these obstacles, two fast monster will attack immediately. In battle, they do not differ from ordinary enemies, they only quickly hope to us.

    Next again narrow corridors. The first battery does not take from the wall, and you can. In the last room, the elevator battery fails, mice will appear and will begin to revive the corpses. We understand with the enemies, insert any other battery, climb the elevator.

    In the elevator cabin, Dr. Terrence Kane will contact us. He will tell that the reason for all the troubles is the artifact "Marker", which was discovered on the planet and delivered to the Ishimour. To stop the invasion of necromorphs, you need to return the "marker" to the place. If we manage to repair the shuttle, then it should be used for this case, and not evacuated.



    Fight with a divided monster, a little further - with Kamikaze. We take from the wall of the power node (23). At the turn we find a shooting gallery, we can practice in shooting. It is necessary to shoot in red armed mannequins, and not shoot in unarmed blue. Endless learning cartridges, modem weapons apply any, but the standard gun is preferable. Total can pass 5 tests:

    1. Kill 10 enemies, 1000 points. Award: Additional Cartridges.

    2. Kill 12 enemies, 1200 points. Reward: Pistol cartridges.

    3. Kill 14 enemies, 1400 points. Award: A middle aid kit.

    4. Kill 16 enemies, 1600 points. Reward: ruby semiconductor.

    5. Kill 20 enemies, 2000 points. Award: Power Node (24).


    Infirmary

    We are going to Lazaret, here a faulty laser scalpel, the rays of which are deadly dangerous. We slow down the laser by the Stasis, quickly run by. On the way to the left of it you can take a large first-aid kit (LARGE MED PACK).

    In the barracks in a dead end, we take the power node (25). In a double corridor, all sorts of monsters will be attached, while the door does not open.


    Engine Room

    We get to the machine compartment, the ships engine see the windows. Raised on the elevator, the powerful stream of fire occurs from above. We need to hide at one of the cylinders, shift it with a kinesis, and to move forward so much. If the cylinder is stuck, we move it. On the way you need to destroy 3 fuses on the pillars. After you need to go back, and pull the cylinder on yourself, which is much more difficult. We pass forward on the other hand, also destroy 3 fuses. After that, the engine will stop. We take singular converter in the center.

    Go down. For the glass we will see the Hammond came running, but it will immediately kill the red brut. This monster has more health than ordinary healthy, but kills the same way. For the victory over him we get diamond semiconductor For 25,000 credits.

    Wairor's ship began to collapse, quickly choose from it. Along the way, we kill a couple of ordinary monsters. Get to the trolley.

    10. Recent days (End of Days)
    Dead Space Passage. Chapter 10.

    Candra reports that I found the location of the shuttle on the ship. But before his launch, you need to collect navigation cards to get to the ground. Search them will be in the residential compartment of the station. In the lobby we see many killed, as with a sacrifice, and near the strange figure, obelisk.


    Sleep Bock C.

    Right enter the open door, find the shower lEVEL 5 SUIT). We can immediately return and buy this costume for 60,000 credits. Further in a dead end, a mini-G Basketball mini-G game. Inside the gym with low gravity lies 1st navigation map. Here we can play this game by turning on the panel on the first platform.

    Z-Ball rules: Of the four holes, the ball flies ahead, we need to catch it and throw it back into any hole. But if we do it right away, we will get only 1 point. When the next ball crashes, several platforms light up, first jump on them, each visit will double the glasses, and after that you can catch and throw the ball back, earn 2, 4 or 8 points. For a minute you need to dial from 12 to 40 points, depending on the level. There are 6 levels in total, for each of them we will receive prizes from lockers:

    1. Just throw the balls. Award: Fuel flameth.

    2. Holes slowly close. Award: A middle aid kit.

    3. Opened only one hole. Reward: a rayan battery.

    4. One hole opens for a while. Award: 3,000 loans.

    5. Holes are opened in turn. Reward: ruby semiconductor.

    6. The holes are opened in turn faster. Award: Power Node (26).


    Mess Hall

    We rise to the second floor. For the glass we see a crazy doctor, he brings a sacrifice - kills a man. We enter the cafeteria, but I do not have time to catch the doctors. We descend under the cafe, we kill two long monsters, from the wall at the table we take the power node (27). Near the woman who took the wall, take key carriage. Climb and go to the next room.


    Sleep Block A.

    In the corridor, we kill the necromorph in the wall. We go on the following floor through the rest room. We will grab and drag the tentacle, shoot in yellowightened. At the end we reach the panel, on the left of the kinesis we move the magnetic lever, it will open the doors on the floor.

    Sleep Block B.

    We enter the depressiveized residential compartment, where oxygen is consumed. There are many monsters, but we do not pay attention to them, there is little time. It is especially not necessary to kill thick monsters, of which leeches will appear, and the flamethrower does not work here. Two times we turn to the right, in a dead-end room we find the power node (28). In the central hall between the rows of Kubinkov will find 2nd navigation map.


    Sleep Bock C.

    We return back to the hall, from there go to the compartment "C", now with the help of the key we can open the door to the bedroom zone. On the way I will see a video of the speech of the doctor.

    In the bedroom on the right we can spend the power knot to open the door, for it various cartridges and a middle aid kit. Kinezis move the bed to go to the cache behind them: I move the bed at the wall, then the bed is across. Entering the intermediate part, we move back the transverse bed, we move two more beds at the right wall, we leave.

    In the cache, we find many corpses and a laughing girl, who immediately suices. Right from it take 3rd navigation map.

    Quickly come back. We will begin to pursue the regenerating monster. Doors will close in the intermediate room, and we will have to run some time with circles from an invulnerable monster until Kendra hack the door. In the lobby, sitting on the elevator.


    Security Station

    We find refuge of Dr. Kane, who helped us. Communicate with him through the glass. He misses us to the direct school.

    Executive Commons

    In the large Red Hall of 3 fat monster. We search for 4 siderooms, in one of them Audio Draw Captain, in one note of the assistant, in the rest - ammunition. We enter double doors.

    Executive Shuttle Bay.

    We get to the starting area of \u200b\u200bthe shuttle, we enter into it, install navigation cards. Dr. Kane asks to wait until he gets here. While we go to the side room and test the engine there.

    But before the doctor, the regenerating monster will turn here. To kill this boss, lure it on the site opposite the shuttle exhaust pipes, shoot him legs, freeze the stasis. Quickly run into the room and turn on the engine again. Monster will remain only ashes and power node (29).

    Dr. Kane will come running and get inside the shuttle. In the room on the second panel open the valve, the shuttle goes down. We go back to Monorails on foot. On the way back, let's see the video like a mad doctor Mercer voluntarily turns into necromorph. We pass to the next room, we kill the bat and mutating the doctor.

    11. Alternative solutions (Alternate Solutions)
    Dead Space. Video list. Chapter 11.

    Cargo Bay.

    Kane by radio will explain that we must find black and red obelisk in the cargo compartment, and download it to the shuttle. Go down to the floor below, first go around the square track all the location, activate the panel. In the far corner on the wall of the power node (30).

    In the center on the conveyor belt we find the obelisk in a glass box. It is necessary to move the obelisk in the conveyor using a kinosis. We are prevented by necromorphs, and when we go out on the central tracks, we are attacked by the giant tentacles, first need to scare them, and then move the cargo.


    FLIGHT LOUNGE.

    We return to the room where the first arrived at the station. If we go on the corridor, where they ran from the first monster, finding a dead end ruby semiconductor and cartridges. Then we go out into the spaceport.


    Hangar Bay.

    Shattl Kane will arrive in the hangar. The loading system does not work, we need to manually download the obelisk to the shuttle. We go to the room to the right, turn off the gravity on the panel, jump down under the ship.

    Secret. In the hangar, we look down to the ditch between the bridges, the Peng statuette is lying on the bottom, we can pull it with a kinesis module and pick up. It is unique, the most expensive thing in the game, we can sell it for 50,000.

    We move the obelisk further, but here there are two fork on the rails, turn them so that the occasion is right. At the first crossroads wech the obelisk in the side deadlock, turn the rails on the left side, move the load to the end.

    We return to the panel on the balcony, we turn on the gravity, the obelisk will plunge. We run to omnounced ladder in the shuttle. But right in front of us Dr. Kane will be killed, and the shuttle will fly away without us. All this arranged Kendra, she admits that was a secret agent, and her goal was to get obelisk. Obelisk himself is a copy created by people for experiments.

    Nicole will come to contact. We rise to it on the top floor of the control room, we take diamond semiconductor. On the screens around the interference and extraterrestrial characters. Nicole will tell you that the shuttle can be returned to the Angar by emergency remote control. The shuttle is really returning, and Kendra catapults on the planet, frightened by our revenge. We enter the returned ship and fly yourself.

    12. Dead Space (Dead Space)
    Dead Space (2008). Chapter 12.

    Landing Pad.

    On the planet we land on the special platform of the earth's colony. All people are killed here. We enter the building, inside there is a semiconductor and everything you need: shop, workbench, saving. In the side locked room, two monsters are locked - bats.

    Under the shuttle, we shift the platform, we load onto the obelisk, we move along the rails. Near the door lacks the battery, go to look for her inside the building, then continue the way.


    Airlock A, Airlock B

    We move the obelisk further, we will interfere with the whole hordes of necromorphs, but we can constantly use the Stasis, there are many panels here for its replenishment. In the block "b" in the right far corner there is a door opened by the power node. In some places you need to carry out obelisk under the bridges, for this, press the panel, freeze the bridges by the Stasis.


    GRAVITY CONTROL

    On the way on the right there are two enemies in the wall. Bridges do not rise, you need to go into a large pipe and turn on electricity. On the way back all the devices will turn on and we will have to pass through moving blades, slow down their stasis. When we return to the main way, it is necessary to fight with the brutal, for the victory over it we get a power knot (31).

    In the next small compartment on the right there is a power node (32).


    Pedestal Chamber.

    We go out on the open surface of the planet. On the way we have a dozen monsters and three tentacles from the cliff. We clean the way and we dotion the obelisk to the cliff.

    The alarm is triggered, running into the glass room on the left. We are locked, and the kendra appears outside, it moves the obelisk back to the shuttle. Also, Kendra throws us a complete video communication from Nicole, in which it sustates. It turns out that the real Nicole has long been dead, and all that we have seen is hallucinations caused by Obelisk. (In the game hid a hint about it: from the first letters of the names of Chapters Dead Space in English there is a phrase: Nicole Is Dead - Nicole is dead).


    Living Space.

    Early surrender. From the glass room we go to the side corridor, overgrown with biomass. On the way, take the power node (33). We return to the first room, last procurement in the store, we spend the power knots, we take more cartridges and first-aid kits. We go to the shuttle.


    Boss: Hive Mind (Mind Roy)

    Kender tried to hide on the shuttle, but at the last moment she was killed by the tentacle of a giant monster. We remain with this monster one to one. This is the leader of necromorphs, being on this planet, he telepathically controlled all monsters.

    1 phase. The boss will hit tentacles from above. We constantly look in his direction, run away left-right. After hitting the boss will bring her mouth towards us. Around the grazing, he has 5 yellow thighs, you need to shoot on them. It is better to spend powerful slow weapons, and the rapid base pistol is left for later.

    2 phase. When they shoot three any outflow, the monster grabs us with a tentacle. In the remaining two outflows will have to shoot on the weight, hanging up the legs. If we do not have time, the boss will swallow us.

    3 phase. Taking yellow dots on time, we remain alive, and fall on the platform. The boss does all the same, but now after a few blows, the Röbra will be revealed, and 5 yellow hearts hang inside them. We use more accurate weapons - the initial gun, gradually shoot the heart.

    When all the hearts are shot, a huge monster will fall off the cliff. Sit down to the shuttle and leave the planet. Inside the last time we watch a video from Nicole, and fly to the ground.

    Congratulations! The whole game is passed!

    For the passage we get the following bonuses:

    Military suit (military armor);

    Backstory Logs (Text messages with the history of the plot);

    50,000 Credits (money from the very beginning);

    10 Power Nodes (10 power nodes);

    Impossible Mode (the maximum complexity opens).

    Questions - Answers

    Question: How to install a lighthouse on an asteroid? What to do with an asteroid? (in the game Dead Space)

    Answer: The asteroid holds 4 beams, we turn off the two of them inside the ship, two more - outside. Only after that we can put the lighthouse. Read more described in chapter 7.


    Question: How to jump in weightlessness? (in the game Dead Space)

    Answer: In the first part of the game, you can not fly in weightlessness, we can only jump over the walls and the ceiling. Aims into the appropriate wall (clamp the right mouse button), we make a jump (press the Ctrl key).


    Question: How to use Stasis? (in the game Dead Space)

    Answer: Aims (clamp the right mouse button), we release the delayed charge (key C).


    Question: How to enable, change the Russian language? (in the game Dead Space)

    Answer: In the licensed version of the game there is no Russian voice acting, only subtitles. To enable subtitles, in the main menu, select the "Settings" stitch - "Sound" - "Subtitles: Enable".


    Question: How to pass the meteor rain? (in the game Dead Space)

    Answer: At the end of the 4th chapter. In order not to die, before the start of the meteoric rain, you need to get up for high shelter, close to it, and wait for a dangerous moment. After that, quickly overlook the next shelter, and so until the end of the way.


    Question: How to remove the power castle? (in the game Dead Space)

    Answer: The game has official storage facilities, locked on the power lock. To open them, you need to spend 1 power node. Nodes can be found in secrets or buy in game stores for 10,000 credits.


    Question: What part is better? (in the game Dead Space)